You are on page 1of 29

Page 1 of 29

SFG 2022 | LEVEL 1 | Test #37 – Solutions |


Q.1) Consider the following statements with respect to cryopreservation and food irradiation:
1. Cryopreservation is a technique of preserving cells and tissue at very low temperature.
2. Food irradiation is the process of passing the food through high energy electrons to eliminate
microorganisms from it.
3. While cryopreservation preserves food without compromising its nutritional content, irradiation
compromises the nutritional quality of food.
Which of the statements given above is/are correct?
a) 1 and 2 only
b) 3 only
c) 2 and 3 only
d) 1, 2 and 3

Ans) a
Exp) Option a is correct
Cryopreservation is a technique in which low temperature is used to preserve the living cells and tissue while
food irradiation is a process that uses radiation to control pests in food and prevent spoilage.
Statement 1 is correct. Cryopreservation is a process that preserves organelles, cells, tissues, or any other
biological constructs by cooling the samples to very low temperatures to maintain their viability. The word
cryo comes from the Greek word "kayos" meaning "frost". It means preservation in a "frozen state".
Cryopreservation uses liquid nitrogen to deep-freeze, and thus preserve, biological materials.
Statement 2 is correct. Food irradiation (the application of ionizing radiation to food) is a technology that
improves the safety and extends the shelf life of foods by reducing or eliminating microorganisms and insects.
During irradiation, gamma rays, x-rays, or high-energy electrons pass through the food, destroying or
inactivating bacteria and viruses that cause foodborne illness.
Statement 3 is incorrect. Preservation of food through cryopreservation delays spoilage and keeps foods
safe by preventing microorganisms from growing and by slowing down the enzyme activity that causes food
to spoil. As the water in the food freezes into ice crystals, it becomes unavailable to those microorganisms
that need it for growth. In contrast, food irradiation does not make foods radioactive, compromise
nutritional quality, or noticeably change the taste, texture, or appearance of food. In fact, any changes made
by irradiation are so minimal that it is not easy to tell if a food has been irradiated. Irradiated food is
considered safe by the World Health Organization (WHO).
Knowledge Base:
Application of food irradiation:
 Effectively eliminate organisms that cause foodborne illness, such as Salmonella and Escherichia coli (E.
coli).
 Destroy or inactivate organisms that cause spoilage and decomposition and extend the shelf life of foods.
 Decreases the need for other pest-control practices that may harm the fruit.
 Inhibit sprouting (e.g., potatoes) and delay ripening of fruit to increase longevity.
 Irradiation can be used to sterilize foods, which can then be stored for years without refrigeration.
Sterilized foods are useful for patients with severely impaired immune systems, such as patients with
AIDS or undergoing chemotherapy.
Source: https://www.sciencedirect.com/topics/engineering/cryopreservation
https://www.ncbi.nlm.nih.gov/pmc/articles/PMC5395684/
https://www.fda.gov/food/buy-store-serve-safe-food/food-irradiation-what-you-need-know
https://www.epa.gov/radiation/what-food-irradiation

Q.2) With reference to the Genetic Engineering Approval Committee (GEAC), consider the following
statements:
1. It is a statutory body under the Ministry of Environment, Forest and Climate Change.
2. It was constituted under the Food Safety and Standards Act, 2006.
3. Its approval is mandatory for the commercial use of genetically modified organisms.
Which of the statements given above is/are correct?
a) 2 only
b) 1 and 2 only
c) 1 and 3 only
d) 3 only

Ans) c
Exp) Option c is correct
The Genetic Engineering Approval Committee (GEAC) was constituted for the appraisal of activities involving
large scale use of hazardous microorganisms and recombinants in research and industrial production from
the environmental angle.

Forum Learning Centre: Delhi - 2nd Floor, IAPL House, 19 Pusa Road, Karol Bagh, New Delhi - 110005 | Patna - 2nd floor, AG Palace, E Boring Canal Road,
Patna, Bihar 800001 | Hyderabad - 1st & 2nd Floor, SM Plaza, RTC X Rd, Indira Park Road, Jawahar Nagar, Hyderabad, Telangana 500020
9821711605 | https://academy.forumias.com | admissions@forumias.academy | helpdesk@forumias.academy
Page 2 of 29

SFG 2022 | LEVEL 1 | Test #37 – Solutions |


Statement 1 is correct. The Genetic Engineering Approval Committee (GEAC) is a statutory body. It is the
apex biotech regulatory body in India and functions under the Ministry of Environment, Forest and Climate
Change (MoEF&CC). It was constituted for regulating the use, manufacture, storage, import and export of
hazardous microorganisms or genetically-engineered organisms and cells in India.
Statement 2 is incorrect. GEAC was constituted under the ‘Rules for the Manufacture, Use /Import /Export
and Storage of Hazardous Microorganisms/Genetically Engineering Organisms or Cells, 1989’ notified under
the Environment (Protection) Act, 1986 (and not Food Safety and Standards Act, 2006). It is chaired by the
Special Secretary/Additional Secretary of MoEF&CC and co-chaired by a representative from the
Department of Biotechnology (DBT).
Statement 3 is correct. The approval of the GEAC is mandatory before genetically modified organisms and
products derived from them can be used commercially. It is also responsible for appraisal of proposals
relating to release of genetically engineered (GE) organisms and products into the environment including
experimental field trials.
Source: https://moef.gov.in/en/project-approvals/geac-clearances/
https://www.isaaa.org/kc/cropbiotechupdate/article/default.asp?ID=6475

Q.3) Other than resistance to pests, what are the prospects for which genetically engineered plants have been
created?
1. To enable them to withstand drought
2. To increase the nutritive value of the produce
3. To enable them to grow and do photosynthesis in spaceships and space stations
4. To increase their shelf life
Select the correct answer using the codes given below:
a) 1 and 2 only
b) 3 and 4 only
c) 1, 2 and 4 only
d) 1, 2, 3 and 4

Ans: c
Exp) Option c is correct.
Genetically modified organisms (GMOs) can be defined as organisms (i.e. plants, animals or microorganisms)
in which the genetic material (DNA) has been altered in a way that does not occur naturally by mating and/or
natural recombination. The technology is often called “modern biotechnology” or “gene technology”,
sometimes also “recombinant DNA technology” or “genetic engineering”. It allows selected individual genes
to be transferred from one organism into another, also between nonrelated species. Foods produced from or
using GM organisms are often referred to as GM foods.
Genetically engineered plants have been created to enable them to withstand drought, to increase the
nutritive value of the produce, to increase their shelf life, etc.
So far, scientists have been unable to create GMO which can grow and do photosynthesis in spaceships and
space stations.
Source: UPSC 2012

Q.4) With reference to Transgenic Animals, consider the following statements:


1. These animals contain foreign genes which are administered deliberately into them.
2. Transgenesis in animals is done through recombinant DNA technology.
3. Pigs can be modified through transgenesis to make them more suitable as organ donors for human
transplant patients.
Which of the statements given above is/are correct?
a) 1 only
b) 1 and 2 only
c) 2 and 3 only
d) 1, 2 and 3

Ans) d
Exp) Option d is correct
A transgenic animal is one whose genome has been altered by the transfer of a gene or genes from another
species or breed.
Statement 1 is correct. A transgenic animal is one that carries a foreign gene that has been deliberately
inserted into its genome. Transgenesis can in principle be used to alter many phenotypic properties that
may increase the value of agriculturally important animals. These include growth rate, fat composition, milk
production, and hair texture.

Forum Learning Centre: Delhi - 2nd Floor, IAPL House, 19 Pusa Road, Karol Bagh, New Delhi - 110005 | Patna - 2nd floor, AG Palace, E Boring Canal Road,
Patna, Bihar 800001 | Hyderabad - 1st & 2nd Floor, SM Plaza, RTC X Rd, Indira Park Road, Jawahar Nagar, Hyderabad, Telangana 500020
9821711605 | https://academy.forumias.com | admissions@forumias.academy | helpdesk@forumias.academy
Page 3 of 29

SFG 2022 | LEVEL 1 | Test #37 – Solutions |


Statement 2 is correct. The foreign gene in transgenic animals is constructed using recombinant DNA
methodology. This involves either combining of DNA from different genomes or the insertion of foreign DNA
into a genome. Example: GloFish are a type of transgenic zebrafish (Danio rerio) that have been modified
through the insertion of a green fluorescent protein (gfp) gene.
Statement 3 is correct. Xenotransplantation is one of the most promising areas for the exploitation of the
transgenic technology, where transgenic pigs are engineered to express immunological characteristics that
would make their organs well tolerated after transplantation into humans. Transgenic animals are also
engineered for increased production of dairy proteins for food, enhanced production of wool or enhanced
disease resistance. In biotechnology, the use of large transgenic animals or ‘bioreactors’ has been attempted,
e.g., for the cost-effective production of high-value pharmaceuticals in the blood or milk of transgenic sheep
or cows.
Source: https://www.sciencedirect.com/topics/agricultural-and-biological-sciences/transgenic-animals
https://whatisbiotechnology.org/index.php/science/summary/transgenic/transgenic-animals-have-
genes-from-other-species-inserted
https://blog.forumias.com/gene-editing-an-analysis/

Q.5) Regarding gene therapy, which of the following statement is incorrect?


a) In this method, a mutated gene is revised, removed, or replaced at the DNA level.
b) This method results in destruction of the problem causing genes.
c) It can be used to treat both inherited genetic diseases and acquired disorders.
d) Viral vectors are used for the transfer of genetic materials.

Ans) a
Exp) Option a is correct
Gene therapy is a technique that modifies a person’s genes (introduction, removal or change in the content
of an individual’s genetic material) to treat or cure disease.
Option a is incorrect. In gene editing, a mutated gene is revised, removed, or replaced at the DNA level. In
gene therapy, the effect of a mutation is offset by inserting a “healthy” version of the gene. Both approaches
may provide a durable benefit to patients, and both gene therapy and gene editing, alone or in combination,
may lend themselves to the development of transformative genomic medicines.
Option b is correct. Gene therapy is used in the replacement of genes that cause medical ill-health. This
method generally destroys the problem causing genes. On the other hand, the goals of gene editing are to
disrupt harmful genes or to repair mutated genes.
Option c is correct. Both inherited genetic diseases and acquired disorders can be treated with gene therapy.
Examples of these disorders are primary immune deficiencies, where gene therapy has been able to fully
correct the presentation of patients, and/or cancer, where the gene therapy is still at the experimental stage.
Option d is correct. Transfer of genetic material is done commonly by using viral vectors that use their own
biological capacities to enter the cell and deposit the genetic material. Several types of viruses, including
retrovirus, adenovirus, adeno-associated virus (AAV), and herpes simplex virus, have been modified in the
laboratory for use in gene therapy applications.
Source: https://www.fda.gov/vaccines-blood-biologics/cellular-gene-therapy-products/what-gene-
therapy
https://www.editasmedicine.com/crispr-gene-editing/
https://www.genome.gov/genetics-glossary/Gene-Therapy
https://blog.forumias.com/icmr-sets-up-a-task-force-on-gene-therapy-research/

Q.6) In the context of the developments in Bioinformatics, the term ‘transcriptome’, sometimes seen in the
news, refers to
a) a range of enzymes used in genome editing
b) the full range of mRNA molecules expressed by an organism
c) the description of the mechanism of gene expression
d) a mechanism of genetic mutations taking place in cells

Ans: b
Exp) Option b is correct.
A transcriptome is the full range of messenger RNA, or mRNA, molecules expressed by an organism.
Transcriptome of a cell or a tissue is the collection of RNAs transcribed in it. In contrast with the genome,
which is characterized by its stability, the transcriptome actively changes.
Messenger RNA (mRNA) is a single-stranded RNA molecule that is complementary
to one of the DNA strands of a gene. Messenger RNA (mRNA) carries the genetic
information copied from DNA in the form of a series of three-base code “words,”
each of which specifies a particular amino acid.

Forum Learning Centre: Delhi - 2nd Floor, IAPL House, 19 Pusa Road, Karol Bagh, New Delhi - 110005 | Patna - 2nd floor, AG Palace, E Boring Canal Road,
Patna, Bihar 800001 | Hyderabad - 1st & 2nd Floor, SM Plaza, RTC X Rd, Indira Park Road, Jawahar Nagar, Hyderabad, Telangana 500020
9821711605 | https://academy.forumias.com | admissions@forumias.academy | helpdesk@forumias.academy
Page 4 of 29

SFG 2022 | LEVEL 1 | Test #37 – Solutions |


Transcriptomics is an emerging and continually growing field in biomarker discovery for use in
assessing the safety of drugs or chemical risk assessment.
Source: UPSC 2016

Q.7) Regarding the Bt Cotton, which of the following statements is incorrect?


a) It is a genetically modified cotton containing pesticide gene from bacteria Bacillus thuringiensis.
b) It is the only approved genetically modified crop for commercialisation in India.
c) It provides effective protection to cotton plants against the bollworm attack.
d) It boosted India’s cotton productivity making it largest cotton exporter in the world.

Ans) d
Exp) Option d is correct
Bt Cotton is a genetically modified cotton grown in India for almost 20 years.
Option a is correct. Bt Cotton is the genetically modified (GM) cotton which contains the pesticide gene from
the bacteria Bacillus thuringiensis (Bt). According to the Ministry of Agriculture, from 2005, adoption of Bt
cotton rose to 81% in 2007, and up to 93% in 2011.
Option b is correct. As of date, the Genetic Engineering Approval Committee (GEAC) has approved the
environmental release of only Bt cotton in India for commercial cultivation. It was approved in 2002. Bt
brinjal is still under moratorium while GM mustard is yet to be approved for commercialisation.
Option c is correct. This pesticide gene in Bt cotton protects the plant from bollworm, thereby increasing
yields and reducing insecticide spraying on the cotton plant.
Option d is incorrect. Bt Cotton transformed India’s cotton sector, as cotton productivity almost doubled in
six years. India’s share in the global production of cotton increased from 12 per cent in 2002 to 25 per cent
by 2014. From a net importer of cotton, India became the second-largest exporter of cotton (and not largest).
Source: https://indianexpress.com/article/opinion/columns/biotechnology-farming-india-7280709/
https://www.thehindu.com/opinion/op-ed/the-twisted-trajectory-of-bt-cotton/article32566091.ece

Q.8) Consider the following statements with respect to the 3D Bio printing:
1. It is an additive manufacturing process that uses cells to print living structures layer by layer.
2. It can mimic the actual micro and macro environment of human tissues.
3. It ensures maximum safety as there is no emission of harmful chemicals in the process.
Which of the statements given above is/are correct?
a) 1 only
b) 1 and 2 only
c) 2 and 3 only
d) 1, 2 and 3

Ans) b
Exp) Option b is correct
Bioprinting is a process where biomaterials such as cells and growth factors are combined to create tissue-
like structures that imitate natural tissues. The technique is widely applicable to the fields of medicine and
bioengineering.
Statement 1 is correct. 3D Bioprinting is a form of additive manufacturing that uses cells and other
biocompatible materials as “inks”, also known as bioinks, to print living structures layer-by-layer which
mimic the behaviour of natural living systems. In essence, bioprinting works in a similar way to conventional
3D printing. A digital model becomes a physical 3D object layer-by-layer. In this instance, however, a living
cell suspension is utilized instead of a thermoplastic or a resin.
Statement 2 is correct. The greatest importance of bioprinting lies in the resulting tissue-like structures that
mimic the actual micro- and macro-environment of human tissues and organs. This is critical in drug
testing and clinical trials, with the potential, for example, to drastically reduce the need for animal trials. The
process could also eradicate the headaches associated with organ donation and transplantation.
Statement 3 is incorrect. There is an emission of harmful chemicals during the bioprinting process. Other
challenges include ensuring the compatibility of materials and cells with the printer and printing processes
in printing vascular network. In addition, it may take longer for vascular networks to develop and mature
throughout the tissue structure than for cells to survive.
Knowledge Base:
Applications of 3D Bioprinting:
 Availability of 3D printed organs helps to solve organ-related issues faster and quicker.
 Development of tissues for pharmaceutical testing, when 3D printed, is a more cost-effective and ethical
option.
 Cosmetic surgery, particularly plastic surgery and skin grafting, also benefits from the technology.
 Bone tissue regeneration as well as prosthetics and dental applications.

Forum Learning Centre: Delhi - 2nd Floor, IAPL House, 19 Pusa Road, Karol Bagh, New Delhi - 110005 | Patna - 2nd floor, AG Palace, E Boring Canal Road,
Patna, Bihar 800001 | Hyderabad - 1st & 2nd Floor, SM Plaza, RTC X Rd, Indira Park Road, Jawahar Nagar, Hyderabad, Telangana 500020
9821711605 | https://academy.forumias.com | admissions@forumias.academy | helpdesk@forumias.academy
Page 5 of 29

SFG 2022 | LEVEL 1 | Test #37 – Solutions |


Source: https://all3dp.com/2/what-is-3d-bioprinting-simply-explained/
https://www.analyticssteps.com/blogs/3d-bioprinting-applications-advantages-and-disadvantages
https://www.sciencedirect.com/science/article/pii/S1002007120305232

Q.9) What is Cas9 protein that is often mentioned in news?


a) A molecular scissors used in targeted gene editing
b) A biosensor used in the accurate detection of pathogens in patients
c) A gene that makes plants pest-resistant
d) A herbicidal substance synthesized in genetically modified crops

Ans: a
Exp) Option a is correct.
Cas9 protein has been in news for its property of being a molecular scissors used in targeted gene editing.
Its main function is to cut DNA and thus alter a cell’s genome. Cas9 plays a vital role in the immunological
defense of certain bacteria against DNA viruses.
The gene editing technique using such DNA scissors is titled “CRIPSR-Cas9”
Source: UPSC 2019

Q.10) Consider the following statements with respect to the comparison between genetically modified
organisms and cloned organisms:
1. While genetically modified organisms have modified DNA with desired traits, a cloned organism is the
genetically identical copy of a biological entity.
2. Unlike cloning, genetic modification does not disrupt the natural process of gene flow.
3. While genetic modification cannot be done naturally, cloning of some organisms occur naturally.
Which of the statements given above is/are correct?
a) 1 and 2 only
b) 2 only
c) 1 only
d) 2 and 3 only

Ans) c
Exp) Option c is correct
Genetically Modified Organisms (GMO) are those plants, bacteria, fungi and animals whose genes have been
altered by manipulation.
Statement 1 is correct. A genetically modified organism (GMO) is an animal, plant, or microbe whose DNA
has been altered using genetic engineering techniques in the laboratory. It is done in order to favour the
expression of desired physiological traits or the generation of desired biological products. In contrast,
cloning describes a number of different processes that can be used to produce genetically identical copies
of a biological entity. The copied material, which has the same genetic makeup as the original, is referred to
as a clone. Researchers have cloned a wide range of biological materials, including genes, cells, tissues and
even entire organisms, such as a sheep.
Statement 2 is incorrect. The production of GMOs imposes high risks to the disruption of ecosystem and
biodiversity because the “better” traits produced from engineering genes can result in the favouring of one
organism. Hence, it can eventually disrupt the natural process of gene flow.
Statement 3 is incorrect. Though much of the controversy around genetically modified organism is driven
by the belief that the process of moving genes from one species to another is “unnatural,” new research
shows some 1 in 20 flowering plants are naturally transgenic.
Dozens of plants, including bananas, peanuts, Surinam cherries, hops, cranberries and tea, contain the
Agrobacterium microbe. The research follows on the heels of the 2015 discovery that sweet potatoes are
naturally transgenic.
Cloning can also occur naturally. In nature, some plants and single-celled organisms, such as bacteria,
produce genetically identical offspring through a process called asexual reproduction. Natural clones, also
known as identical twins, occur in humans and other mammals. These twins are produced when a fertilized
egg splits, creating two or more embryos that carry almost identical DNA. Identical twins have nearly the
same genetic makeup as each other, but they are genetically different from either parent.
Source: https://www.nationalgeographic.org/encyclopedia/genetically-modified-organisms/
https://www.agrifarming.in/gm-crops-advantages-and-disadvantages
https://allianceforscience.cornell.edu/blog/2019/10/many-plants-naturally-gmo-research-finds/
https://www.dw.com/en/when-gmo-comes-naturally/a-18399170
https://www.genome.gov/about-genomics/fact-sheets/Cloning-Fact-Sheet

Forum Learning Centre: Delhi - 2nd Floor, IAPL House, 19 Pusa Road, Karol Bagh, New Delhi - 110005 | Patna - 2nd floor, AG Palace, E Boring Canal Road,
Patna, Bihar 800001 | Hyderabad - 1st & 2nd Floor, SM Plaza, RTC X Rd, Indira Park Road, Jawahar Nagar, Hyderabad, Telangana 500020
9821711605 | https://academy.forumias.com | admissions@forumias.academy | helpdesk@forumias.academy
Page 6 of 29

SFG 2022 | LEVEL 1 | Test #37 – Solutions |


Q.11) With reference to the stem cells, consider the following statements:
1. Stem cells are unspecialized cells capable of dividing and renewing themselves.
2. Unlike embryonic stem cells, adult stem cells can become any cell in the body.
3. Stem cell therapy is a non-surgical procedure of repairing damaged tissues.
4. It has no side effects as it induces the body to recover through natural healing.
Which of the statements given above is/are correct?
a) 2 and 3 only
b) 1, 3 and 4 only
c) 2 and 4 only
d) 1, 2, 3 and 4

Ans) b
Exp) Option b is correct
Stem cells are the body's raw materials — cells from which all other cells with specialized functions are
generated. They are basically those cells which may replicate themselves many times, or proliferate.
Statement 1 is correct. Stem cells are undifferentiated biological cells that can differentiate into specialized
cells and can divide to produce more stem cells. Stem cells are distinguished from other cell types by two
important characteristics. First, they are unspecialized cells capable of dividing and renewing themselves
through cell division, sometimes after long periods of inactivity. Second, under certain physiologic or
experimental conditions, they can be induced to become tissue- or organ-specific cells with special
functions. In some organs, such as the gut and bone marrow, stem cells regularly divide to repair and replace
worn out or damaged tissues.
Statement 2 is incorrect. Embryonic stem cells are derived from embryos that develop from eggs that have
been fertilized in vitro. An adult stem cell is thought to be an undifferentiated cell, found among differentiated
cells in a tissue or organ. Embryonic are pluripotent stem cells i.e. they can become any type of cell in the
body. This versatility allows embryonic stem cells to be used to regenerate or repair diseased tissue and
organs. Compared with embryonic stem cells, adult stem cells have a more limited ability to give rise to
various cells of the body. Other types of stem cells are foetal stem cells (taken from aborted foetal tissue)
and umbilical stem cells (taken from umbilical cord).
Statement 3 is correct. Stem cell therapy is a type of treatment option that uses a patient’s own stem cells
to repair damaged tissue and repair injuries. It is a non-surgical procedure with no incisions of the body.
Stem cells are usually taken from one of the two areas in the patient’s body: bone marrow or adipose (fat)
tissue in their upper thigh/abdomen. It is used to treat more than 80 disorders including neuromuscular and
degenerative disorders. Eg. Bone-marrow transplant is used in Leukemia (blood cancer), sickle-cell anaemia,
and immunodeficiency disorders.
Statement 4 is correct. Unlike most of the medical treatments, stem cell therapy comes with no side effects.
It also prevents infections and reduces the risk of complications through natural healing. Once the stem cells
are injected at the site of injury, they stimulate the growth of new cells, which repair and heal the damaged
tissue.
Source: https://www.oatext.com/What-do-we-know-about-stem-cells-and-application-in-
Pharmacology.php
https://www.mayoclinic.org/tests-procedures/bone-marrow-transplant/in-depth/stem-cells/art-
20048117
https://blog.forumias.com/answereddescribe-briefly-what-stem-cell-therapy-is-and-what-advantages-
it-has-over-other-treatments/

Q.12) Which of the following are the reasons for the occurrence of multi-drug resistance in microbial
pathogens in India?
1. Genetic predisposition of some people
2. Taking incorrect doses of antibiotics to cure diseases
3. Using antibiotics in livestock farming
4. Multiple chronic diseases in some people
Select the correct answer using the code given below.
a) 1 and 2 only
b) 2 and 3 only
c) 1, 3 and 4 only
d) 2, 3 and 4 only

Ans: b
Exp) Option b is correct.
 Multi-drug resistance is not a hereditary problem.

Forum Learning Centre: Delhi - 2nd Floor, IAPL House, 19 Pusa Road, Karol Bagh, New Delhi - 110005 | Patna - 2nd floor, AG Palace, E Boring Canal Road,
Patna, Bihar 800001 | Hyderabad - 1st & 2nd Floor, SM Plaza, RTC X Rd, Indira Park Road, Jawahar Nagar, Hyderabad, Telangana 500020
9821711605 | https://academy.forumias.com | admissions@forumias.academy | helpdesk@forumias.academy
Page 7 of 29

SFG 2022 | LEVEL 1 | Test #37 – Solutions |


 Overuse and misuse of antimicrobial agents is the single most important cause of development of
resistance.
 Anti-biotics are also used indiscriminately as growth promoters in animals or used to prevent diseases in
healthy animals. The antibiotics used in livestock are ingested by humans when they consume food, this
transfers resistant bacteria to humans through consumption or environment.
 There is also no conclusive evidence to prove that presence of multiple chronic diseases is a reason for
presence for occurrence of multi-drug resistance in India.
Source: UPSC 2019

Q.13) Consider the following statements with respect to the difference between Pressurised Heavy Water
Reactors (PHWRs) and Fast Breeder Reactors (FBRs):
1. While PHWR uses natural uranium as fuel, a fast breeder reactor uses thorium to produce electricity.
2. While PHWRs have a neutron moderator, it is not present in FBR.
3. Unlike FBR, PHWRs produce less radioactive waste as they use heavy water as coolant.
Which of the statements given above is/are correct?
a) 1 and 3 only
b) 2 only
c) 2 and 3 only
d) 1, 2 and 3

Ans) b
Exp) Option b is correct
Nuclear reactors generate energy through fission, the process by which an atomic nucleus splits into two or
more smaller nuclei. During fission, a small amount of mass is converted into energy, which can be used to
power a generator to create electricity.
Statement 1 is incorrect. The Pressurised Heavy Water Reactors (PHWRs) uses natural Uranium as fuel.
PHWRs not only produce energy from natural uranium but also produce fissile plutonium (Pu)-239. The Fast
Breeder Reactors (FBRs) use plutonium-239 (and not thorium) for generating electricity. Once a sufficient
amount of plutonium-239 is built up, thorium will be used in the reactor, to produce Uranium-233.
Statement 2 is correct. PHWR uses heavy water (deuterium oxide D2O) as its coolant and neutron
moderator. The heavy water coolant is kept under pressure, allowing it to be heated to higher temperatures
without boiling. The heavy water creates greatly enhanced neutron economy, allowing the reactor to operate
without fuel-enrichment facilities (offsetting the additional expense of the heavy water) and enhancing the
ability of the reactor to make use of alternate fuel cycles. FBRs do not have a neutron moderator, and use
less-moderating coolants such as liquid sodium, so its neutrons remain high-energy. Breeder reactors use a
small core, which is important to sustain chain reactions.
Statement 3 is incorrect. The increased rate of fuel movement through the PHWR results in higher volumes
of spent fuel. However, fast reactors have the potential to produce less radioactive waste because fuel is
highly enriched in fissile material. There is no need of large quantity of fuel materials for the annual external
feed in FBR and thus eliminates the need for large capacity waste storage spaces with complex construction
features.
Source: https://www.scientificamerican.com/article/how-do-fast-breeder-react/
https://blog.forumias.com/answered-give-an-account-of-the-growth-and-development-of-nuclear-
science-and-technology-in-india-what-is-the-advantage-of-fast-breeder-reactor-programme-in-india/

Q.14) Consider the following statements:


1. Dark matter does not interact with the electromagnetic forces including light.
2. Dark energy causes the universe to expand at an accelerated rate.
3. Dark energy and Dark matter together constitute less than 10% of the universe.
Which of the statements given above is/are correct?
a) 1 only
b) 1 and 2 only
c) 3 only
d) 1, 2 and 3

Ans) b
Exp) Option b is correct.
Dark matter is a hypothetical form of matter thought to account for approximately 85% of the matter in the
universe. Dark energy is a hypothetical form of energy whose negative pressure counteracts gravity and is
assumed to be responsible for the universe expanding at an accelerating rate.
Statement 1 is correct.

Forum Learning Centre: Delhi - 2nd Floor, IAPL House, 19 Pusa Road, Karol Bagh, New Delhi - 110005 | Patna - 2nd floor, AG Palace, E Boring Canal Road,
Patna, Bihar 800001 | Hyderabad - 1st & 2nd Floor, SM Plaza, RTC X Rd, Indira Park Road, Jawahar Nagar, Hyderabad, Telangana 500020
9821711605 | https://academy.forumias.com | admissions@forumias.academy | helpdesk@forumias.academy
Page 8 of 29

SFG 2022 | LEVEL 1 | Test #37 – Solutions |


The mass of the universe is made up of material that scientists cannot directly observe which is known as
dark matter. Unlike normal matter, dark matter does not interact with the electromagnetic force. This
means it does not absorb, reflect or emit light, making it extremely hard to spot. In fact, researchers have
been able to infer the existence of dark matter only from the gravitational effect it seems to have on visible
matter.
Statement 2 is correct.
The dark energy is evenly distributed throughout the universe. It does not have any local gravitational effects,
but rather a global effect on the universe as a whole. This leads to a repulsive force, which tends to accelerate
the expansion of the universe. Thus, Dark energy is the name given to the mysterious force that's causing
the rate of expansion of our universe to accelerate over time, rather than to slow down. The rate of
expansion and its acceleration can be measured by observations based on the Hubble law. These
measurements, together with other scientific data, have confirmed the existence of dark energy and provide
an estimate of just how much of this mysterious substance exists.
Statement 3 is incorrect.
It turns out that roughly 68% of the universe is dark energy. Dark matter makes up about 27%. The rest -
everything on Earth, everything ever observed with all of our instruments, all normal matter - adds up to
less than 5% of the universe.
Source: https://science.nasa.gov/astrophysics/focus-areas/what-is-dark-energy

Q.15) With reference to carbon nanotubes, consider the following statements:


1. They can be used as carriers of drugs and antigens in the human body.
2. They can be made into artificial blood capillaries for an injured part of the human body.
3. They can be used in biochemical sensors.
4. Carbon nanotubes are biodegradable.
Which of the statements given above are correct?
a) 1 and 2 only
b) 2, 3 and 4 only
c) 1, 3 and 4 only
d) 1, 2, 3 and 4

Ans: c
Exp) Option c is correct.
Statement 1 is correct: Carbon nanotubes (CNTs) are cylindrical molecules that consist of rolled-up sheets
of single-layer carbon atoms (graphene). Their hollow interior can be filled with various nanomaterials,
separating and shielding them from the surrounding environment - a property that is extremely useful for
nanomedicine applications like drug delivery.
Statement 2 is incorrect: Carbon Nanotubes cannot be made into artificial blood capillaries for an injured
part of human body.
Statement 3 is correct: CNTs can serve as support for immobilization of biomolecules at their surface,
and combine several exceptional physical, chemical, electrical, and optical characteristics properties which
make them one of the best suited biosensors for the transduction of signals associated with the
recognition of analytes, metabolites, or disease biomarkers.
Statement 4 is correct: Carbon nanotubes are biodegradable, Bacteria, fungi and other multiple types of
microbes have the ability to degrade carbon nanotubes (CNTs), graphene (GRA), and their derivatives and in
the future more species with this ability will be found.
Source: UPSC 2020

Q.16) Which of the following statements in context of satellite launch vehicles in India is/are correct?
1. Geosynchronous Satellite Launch Vehicle (GSLV) has the capability to put a heavier payload in the orbit as
compared to the Polar Satellite Launch Vehicle (PSLV).
2. PSLVs are designed mainly to deliver earth observation or remote sensing satellites.
3. GSLVs are designed for launching mainly communication satellites into higher Geo synchronous orbit.
4. Both PSLV and GSLV are three-stage launch vehicles.
Select the correct answer using the code given below:
a) 1, 2 and 3 only
b) 1 and 4 only
c) 2 and 3 only
d) 1, 2, 3 and 4

Ans) a
Exp) Option a is correct.

Forum Learning Centre: Delhi - 2nd Floor, IAPL House, 19 Pusa Road, Karol Bagh, New Delhi - 110005 | Patna - 2nd floor, AG Palace, E Boring Canal Road,
Patna, Bihar 800001 | Hyderabad - 1st & 2nd Floor, SM Plaza, RTC X Rd, Indira Park Road, Jawahar Nagar, Hyderabad, Telangana 500020
9821711605 | https://academy.forumias.com | admissions@forumias.academy | helpdesk@forumias.academy
Page 9 of 29

SFG 2022 | LEVEL 1 | Test #37 – Solutions |


Geosynchronous Satellite Launch Vehicle (GSLV) is an expendable space launch vehicle designed, developed,
and operated by the Indian Space Research Organisation (ISRO) to launch satellites and other space objects
into Geosynchronous Transfer Orbits.
Polar Satellite Launch Vehicle (PSLV) is the third generation launch vehicle of India. It is the first Indian launch
vehicle to be equipped with liquid stages.
Statement 1 is correct. GSLV has the capability to put a heavier payload in the orbit than the Polar Satellite
Launch Vehicle (PSLV).
PSLV can carry satellites up to a total weight of 2000 kg into space and reach up to an altitude of 600-900
km. GSLV can carry weight up to 5,000 kg and reach up to 36,000 km.
Statement 2 and 3 is correct. PSLV is designed mainly to deliver earth observation or remote sensing
satellites. Whereas, GSLV has been designed for launching communication satellites. GSLV delivers
satellites into a higher elliptical orbit, Geosynchronous Transfer Orbit (GTO) and Geosynchronous Earth
Orbit (GEO).
Statement 4 is incorrect. PSLV is the third-generation launch vehicle of India with 4 stages while GSLV is a
three-stage launcher with strap-on motors.
Source: https://www.isro.gov.in/launchers/pslv
https://www.business-standard.com/about/what-is-gslv

Q.17) With reference to the difference between the Aluminium-Air batteries and Lithium-ion batteries,
consider the following statements:
1. Aluminium-Air batteries have a higher energy density than Lithium-ion batteries.
2. Aluminium-Air batteries cannot be recharged like Lithium-ion batteries.
3. Aluminium-Air batteries may be cheaper in cost than Lithium-ion batteries.
Which of the statements given above is/are correct?
a) 1 and 3 only
b) 2 only
c) 1 and 2 only
d) 1, 2 and 3

Ans) d
Exp) Option d is correct.
An aluminium-air cell consists of a pair of negative and positive electrodes and an electrolyte. In this case,
the anode is formed by aluminium, while the ‘air-breathing’ cathode delivers oxygen from the ambient air.
The electrolyte is water-based and is essentially a solution of potassium hydroxide or sodium hydroxide.
Statement 1 is correct. Aluminium-Air batteries have the higher energy density than Lithium-ion batteries.
Among all metal–air batteries, Al–air batteries hold great promise for future large-scale energy applications
due to their lowest cost and high theoretical specific capacity. It has one of the highest energy densities for
a battery.
Statement 2 is correct. One of the key downsides of aluminium-air batteries is that they cannot be
recharged like lithium-ion batteries. The battery pack is much lighter as compared to lithium-ion battery.
It thus requires battery swapping, not re-charging, so it faces infrastructure and logistical challenges. Large
scale use of aluminium-air battery-based vehicles would require the wide availability of battery swapping
stations.
Statement 3 is correct. Aluminium-air based batteries are expected to be significantly cheaper than
lithium-ion batteries. Thereby reducing the cost of electric vehicle usage and boosting electric vehicle
adoption in the country. Aluminium-air battery-based electric vehicles are expected to offer much greater
range of 400 km or more per battery compared to lithium-ion batteries which currently offer a range of
150-200 kilometres per full charge.
Knowledge Base:
The aluminium, oxygen and water undergo an electrochemical reaction to produce aluminium hydroxide and
electrical energy, which is used to power the motor in an EV.
Aluminium-air batteries utilise oxygen in the air which reacts with an aluminium hydroxide
solution to oxidise the aluminium and produce electricity.
The battery works by tapping electricity generated when aluminium plates react with oxygen in the air.
Source: https://www.business-standard.com/article/automobile/india-to-invest-in-aluminium-battery-
in-evs-as-it-pushes-against-china-121070200097_1.html

Q.18) Consider the following statements:


1. Genetic changes can be introduced in the cells that produce eggs or sperms of a prospective parent.
2. A person’s genome can be edited before birth at the early embryonic stage.
3. Human induced pluripotent stem cells can be injected into the embryo of a pig.
Which of the statements given above is/are correct?

Forum Learning Centre: Delhi - 2nd Floor, IAPL House, 19 Pusa Road, Karol Bagh, New Delhi - 110005 | Patna - 2nd floor, AG Palace, E Boring Canal Road,
Patna, Bihar 800001 | Hyderabad - 1st & 2nd Floor, SM Plaza, RTC X Rd, Indira Park Road, Jawahar Nagar, Hyderabad, Telangana 500020
9821711605 | https://academy.forumias.com | admissions@forumias.academy | helpdesk@forumias.academy
Page 10 of 29

SFG 2022 | LEVEL 1 | Test #37 – Solutions |


a) 1 only
b) 2 and 3 only
c) 2 only
d) 1, 2 and 3

Ans: d
Exp) Option d is correct.
Genome editing can be employed to make changes to specific parts of a genome through Somatic cell therapy
(after birth) and germline genome editing (before birth). The embryonic cells (blastomeres) are totipotent
cells.
Pluripotent cells have the ability to undergo self-renewal and to give rise to all cells of the tissues of the body.
Inducing such human cells in a pig we can cultivate a human like heart, lung, liver, spleen etc.
Source: UPSC 2020

Q.19) In recent times, hydrogen has been increasingly mooted as an alternate source of energy. What is/are
the advantage/advantages of using hydrogen as fuel?
1. Hydrogen can be generated from renewable energy resources.
2. As a fuel, it is more efficient than petrol.
3. The process of extracting hydrogen is highly energy efficient.
4. Use of hydrogen can lead to zero vehicular emissions.
Select the correct answer using the code given below:
a) 1, 2 and 4 only
b) 1 and 4 only
c) 2 and 4 only
d) 1, 2, 3 and 4

Ans) a
Exp) Option a is correct.
The finance minister in the Union budget for 2020-21 formally announced the National Hydrogen Mission
which aims for generation of hydrogen from green power resources.
Statement 1 is correct.
Depending on the nature of the method of its extraction, hydrogen is categorised into three categories,
namely, grey, blue and green. Hydrogen produced from fossil fuels is called grey hydrogen; this constitutes
the bulk of the hydrogen produced today. Hydrogen generated from fossil fuels with carbon capture and
storage options is called blue hydrogen; hydrogen generated entirely from renewable power sources is
called green hydrogen. In the last process, electricity generated from renewable energy is used to split
water into hydrogen and oxygen.
Statement 2 is correct. Hydrogen is two-three times more efficient than burning petrol. The energy in 1
kilogram of hydrogen gas is about the same as the energy in 2.8 kilograms of petrol.
Hydrogen has a high energy density as compared to other fuels, thus it produces more energy in lesser
weight due to which it can prove to be a viable option for heavy vehicles covering long routes in the future.
The refuelling time required for hydrogen is also lesser when compared to electric vehicles and vehicles
running on conventional fuels.
Statement 3 is incorrect. Although hydrogen is a clean molecule, the process of extracting it is energy-
intensive. For instance, to extract hydrogen from water electrolysis is to be performed which itself uses
electricity. Electrolysis is the process of using electricity to split water into hydrogen and oxygen.
Statement 4 is correct. Hydrogen is one of the cleanest fuels, which on being burnt in air produces only
water as a by-product and no carbon-based emissions are released, thereby leading to zero vehicular
emissions.
Knowledge base: Green hydrogen has specific advantages. One, it is a clean burning molecule, which can
decarbonise a range of sectors including iron and steel, chemicals, and transportation. Two, renewable
energy that cannot be stored or used by the grid can be channeled to produce hydrogen.
Source: https://indianexpress.com/article/explained/hydrogen-as-the-alternative-fuel-quixplained-
7198064/
https://indianexpress.com/article/explained/electric-vehicle-technology-hydrogen-car-national-
hydrogen-mission-7193217/
https://energy.economictimes.indiatimes.com/energy-speak/hydrogen-as-a-transportation-fuel-in-
india/3789

Q.20) Consider the following statements about the various techniques of ‘Assisted Reproductive Technology’
(ART):
1. Invitro Fertilization cannot be performed if the fallopian tubes are blocked or absent.

Forum Learning Centre: Delhi - 2nd Floor, IAPL House, 19 Pusa Road, Karol Bagh, New Delhi - 110005 | Patna - 2nd floor, AG Palace, E Boring Canal Road,
Patna, Bihar 800001 | Hyderabad - 1st & 2nd Floor, SM Plaza, RTC X Rd, Indira Park Road, Jawahar Nagar, Hyderabad, Telangana 500020
9821711605 | https://academy.forumias.com | admissions@forumias.academy | helpdesk@forumias.academy
Page 11 of 29

SFG 2022 | LEVEL 1 | Test #37 – Solutions |


2. Gamete Intrafallopian Transfer involves combining eggs and sperms outside the human body.
3. In Zygote Intrafallopian Transfer, eggs are fertilized with the male sperm outside the human body.
4. Unlike Gamete Intrafallopian Transfer, Invitro Fertilization does not result in "biologically related" child.
Which of the above statements is/are correct?
a) 1, 2 and 3 only
b) 1 and 4 only
c) 2, 3 and 4 only
d) 2 and 3 only

Ans) d
Exp) Option d is correct.
Assisted reproductive technology (ART) refers to fertility treatments and procedures that can help with
difficulties or an inability to conceive children.
Assisted Reproductive Technology (ART) includes In vitro fertilization-embryo transfer (IVF-ET), gamete
intrafallopian transfer (GIFT), zygote intrafallopian transfer (ZIFT), and frozen embryo transfer (FET).

Statement 1 is incorrect.
In Invitro Fertilization, eggs are surgically removed from the ovary and fertilised with sperm outside the body
in a Petri dish. The fertilized eggs (embryos) are then placed in the women's uterus, thus bypassing the
fallopian tubes. This technique is specially beneficial where women have blocked or absent fallopian tubes,
or where men have low sperm counts.
In the female reproductive tract, there is one ovary and one fallopian tube on each side of the uterus.
Fertilization of eggs take place here.
Statement 2 is correct.
Gamete Intrafallopian Transfer is a three-step procedure that involves removing the eggs, combining them
with sperm, and immediately placing them in the fallopian tubes, where the egg is fertilized. GIFT is an option
only if the woman has a healthy uterus and fallopian tubes. As there is no IVF procedure, a person does not
have to choose which embryo to transfer.
A specific advantage of the GIFT procedure is that the embryos reaching the uterine cavity at a potentially
more appropriate (i.e., later) stage of development than with IVF-ET.
Statement 3 is correct.
In Zygote Intrafallopian Transfer (ZIFT) eggs are removed and fertilized with the male partner’s sperm in
vitro. Then, zygotes (fertilized eggs) are inserted into the fallopian tubes. Success rates for ZIFT and GIFT are
about the same as for IVF.
A frozen embryo transfer is the process of using a frozen embryo from a previous IVF cycle. It’s gaining
popularity due to its safety and pregnancy success rates. During this procedure, a woman undergoes the
standard IVF procedure, but instead of transferring the embryos back into the uterus in the same cycle, they
are frozen for use in a future cycle.
Statement 4 is incorrect. All the above techniques result in a "biologically related" child.
Source: https://www.medicalnewstoday.com/articles/assisted-reproductive-technology#preparation
https://www.reproductivefacts.org/news-and-publications/patient-fact-sheets-and-
booklets/documents/fact-sheets-and-info-booklets/assisted-reproductive-technologies-booklet/

Q.21) In the context of recent advances of human reproductive technology, “Pronuclear Transfer” is used for
a) fertilization of egg in vitro by the donor sperm
b) genetic modification of sperm producing cells
c) development of stem cells into functional embryos
d) prevention of mitochondrial diseases in offspring

Forum Learning Centre: Delhi - 2nd Floor, IAPL House, 19 Pusa Road, Karol Bagh, New Delhi - 110005 | Patna - 2nd floor, AG Palace, E Boring Canal Road,
Patna, Bihar 800001 | Hyderabad - 1st & 2nd Floor, SM Plaza, RTC X Rd, Indira Park Road, Jawahar Nagar, Hyderabad, Telangana 500020
9821711605 | https://academy.forumias.com | admissions@forumias.academy | helpdesk@forumias.academy
Page 12 of 29

SFG 2022 | LEVEL 1 | Test #37 – Solutions |


Ans: d
Exp) Option d is correct.
Mitochondrial Gene Therapy is a medical technique in which defective mitochondria carried by a woman is
replaced with the healthy mitochondria of a donor. The two most common techniques in mitochondrial gene
therapy are maternal spindle transfer and pronuclear transfer.
Source: UPSC 2020

Q.22) With reference to various types of vaccines, consider the following statements:
1. Unlike DNA vaccines, RNA vaccines instruct cells in our body to produce a protein that induces an immune
response.
2.RNA vaccines can be synthesised chemically without the need for any bacteria or cells.
3. Unlike DNA vaccines, RNA vaccines have zero risk of integrating with our own genetic material.
Which of the above statements is/are correct?
a) 1 only
b) 1 and 2 only
c) 3 only
d) 2 and 3 only

Ans) d
Exp) Option d is correct.
Vaccines train your immune system to create antibodies, just as it does when it's exposed to a disease.
Vaccination is a simple, safe, and effective way of protecting you against harmful diseases, before you come
into contact with them. It uses your body’s natural defenses to build resistance to specific infections and
makes your immune system stronger.
Statement 1 is incorrect. DNA and RNA vaccines both instruct cells in our body to produce a protein that
induces an immune response.
Once inserted into host cells, the genetic material is read by the cell’s own protein-making machinery and
used to manufacture antigens, which then trigger an immune response.
Both DNA and RNA vaccines are relatively easy to produce, but the manufacturing process differs slightly
between them.
Statement 2 is correct. Unlike DNA vaccines, RNA vaccines can be synthesised chemically, without the need
for any bacteria or cells.
RNA vaccines are easier to synthesize because this can be done chemically, from a template in the lab,
without the need for any bacteria or cells. In both cases, vaccines for different antigens could be
manufactured within the same facilities, further reducing costs. This is not possible for most conventional
vaccines.
Statement 3 is correct. While mRNA vaccines cannot affect our genes, this is a potential risk with DNA
vaccines. RNA vaccines encode the antigen of interest in messenger RNA (mRNA) or self-amplifying RNA
(saRNA). Because of its transitory nature, there is zero risk of it integrating with our own genetic material.
Source: https://www.gavi.org/vaccineswork/what-are-nucleic-acid-vaccines-and-how-could-they-be-
used-against-covid-19

Q.23) Which of the following statements about ‘light amplification by the stimulated emission of radiation’
(Laser) technology is incorrect?
a) Laser is a device that stimulates atoms or molecules to emit light.
b) Helium-neon lasers were the first lasers with broad commercial applications.
c) Lasers do not occur in nature and have to be created artificially.
d) Lasers produce a narrow beam of light waves all having very different wavelengths.

Ans) d
Exp) Option d is correct.
Laser is an acronym for “light amplification by the stimulated emission of radiation.”
Statement a is correct. Laser is a device that stimulates atoms or molecules to emit light at particular
wavelengths. It amplifies that light, typically producing a very narrow beam of radiation. The emission
generally covers an extremely limited range of visible, infrared, or ultraviolet wavelengths.
Statement b is correct. Helium-neon lasers were the first lasers with broad commercial applications.
Because they could be adjusted to generate a visible red beam instead of an infrared beam, they found
immediate use projecting straight lines for alignment, surveying, construction, and irrigation. Soon eye
surgeons were using pulses from ruby lasers to weld detached retinas back in place without cutting into the
eye.
Statement c is correct. Lasers do not occur in nature. However, we have figured ways to artificially create
this special type of light.

Forum Learning Centre: Delhi - 2nd Floor, IAPL House, 19 Pusa Road, Karol Bagh, New Delhi - 110005 | Patna - 2nd floor, AG Palace, E Boring Canal Road,
Patna, Bihar 800001 | Hyderabad - 1st & 2nd Floor, SM Plaza, RTC X Rd, Indira Park Road, Jawahar Nagar, Hyderabad, Telangana 500020
9821711605 | https://academy.forumias.com | admissions@forumias.academy | helpdesk@forumias.academy
Page 13 of 29

SFG 2022 | LEVEL 1 | Test #37 – Solutions |


Statement d is incorrect. Lasers produce a narrow beam of light in which all of the light waves have similar
wavelengths. This is why laser beams are very narrow, very bright, and can be focused into a very tiny spot.
Source: https://www.britannica.com/technology/laser
https://spaceplace.nasa.gov/laser/en/

Q.24) What is the difference between Bluetooth and Wi-Fi devices?


a) Bluetooth uses 2.4 GHz radio frequency band, whereas Wi-Fi can use 2.4 GHz or 5 GHz frequency band
b) Bluetooth is used for Wireless Local Area Networks (WLAN) only, whereas Wi-Fi is used for Wireless Wide
Area Networks (WWAN) only
c) When information is transmitted between two devices using Bluetooth technology, the devices have to be
in the line of sight of each other, but when Wi-Fi technology is used the devices need not be in the line of
sight of each other
d) The statements (a) and (b) given above are correct in this context

Ans: a
Exp) Option a is correct.
Bluetooth uses short-wavelength radio waves of a frequency range between 2.4 and 2.485 GHz. Bluetooth
enables one to create a personal area network wherein multiple devices talk to each other wirelessly via
Bluetooth—a typical usage is home control automation systems.
Wi-Fi can use 2.4 GHz or 5 GHz frequency band - A 2.4 GHz connection travels farther at lower speeds,
while 5 GHz frequencies provide faster speeds at shorter range. Both Bluetooth and Wifi need not be in line
of sight for information transmission.
Source: UPSC 2011

Q.25) A superconductor is a material that achieves superconductivity. Which among the following best
describes the term ‘superconductivity’?
a) It is a state of matter that has very low electrical resistance and allows magnetic fields to penetrate.
b) It is a state of matter that has high electrical resistance and does not allow magnetic fields to penetrate.
c) It is a state of matter that has high electrical resistance and allows magnetic fields to penetrate.
d) It is a state of matter that has no electrical resistance and does not allow magnetic fields to penetrate.

Ans) d
Exp) Option d is correct.
Superconducting materials show zero electrical resistance at low temperatures. It allows them to conduct
'supercurrents' without heat dissipation.
Option d is correct. A superconductor is a material that achieves superconductivity. It is a state of matter
that has no electrical resistance and does not allow magnetic fields to penetrate. An electric current in a
superconductor can persist indefinitely. Superconductivity can only typically be achieved at very low
temperatures.
Superconductors have a wide variety of everyday applications, from MRI machines to super-fast maglev
trains that use magnets to levitate the trains off the track to reduce friction.
Superconducting electromagnets can be used in maglev trains, experimental nuclear fusion reactors and
high-energy particle accelerator laboratories. Superconductors can also be used to power railguns and
coilguns, cell phone base stations, fast digital circuits and particle detectors.
Source: https://www.livescience.com/superconductor

Q.26) With reference to communications technologies like 5G and 6G, consider the following statements:
1. 5G has a new interface, which uses millimeter wave spectrum.
2. Unlike 5G that works at high frequencies only, 6G is divided into three different bands- the low, the mid
and the high frequencies.
3. 6G seeks to utilize the terahertz band of frequency which is currently unutilized.
Which of the above statements given above is/are correct?
a) 1 and 2 only
b) 2 only
c) 1 and 3 only
d) 3 only

Ans) c
Exp) Option c is correct.
Each generation of wireless broadband is defined as a set of telephone network standards that describe the
technological implementation of the system.

Forum Learning Centre: Delhi - 2nd Floor, IAPL House, 19 Pusa Road, Karol Bagh, New Delhi - 110005 | Patna - 2nd floor, AG Palace, E Boring Canal Road,
Patna, Bihar 800001 | Hyderabad - 1st & 2nd Floor, SM Plaza, RTC X Rd, Indira Park Road, Jawahar Nagar, Hyderabad, Telangana 500020
9821711605 | https://academy.forumias.com | admissions@forumias.academy | helpdesk@forumias.academy
Page 14 of 29

SFG 2022 | LEVEL 1 | Test #37 – Solutions |


Statement 1 is correct. 5G has a new interface, which uses millimeter wave spectrum (30-300 GHz), which
enables more devices to be used within the same geographic area; 4G can support about 4,000 devices per
square kilometre, whereas 5G will support around one million. This means more Netflix streaming, voice
calls and You Tube carried, without interruption, over the limited air space.
Statement 2 is incorrect. There are three broad categories of 5G bands– the low, the mid and the high and
all three have their own list of pros and cons. The 6G spectrum will land somewhere around 100GHz to 3THz.
This high frequency will be capable of delivering a wireless internet of 5gpbs to 10gbps, it's a massive jump
from a 5G network which is expected to deliver 500mbps to 1gbps speed.
Statement 3 is correct. 6G (sixth-generation wireless) is the successor to 5G cellular technology.
It seeks to utilize the terahertz band of frequency which is currently unutilized.
Terahertz waves fall between infrared waves and microwaves on the electromagnetic spectrum. 6G speed
is expected to be 100 times faster than 5G with enhanced reliability and wider network coverage. 6G internet
will be to support one microsecond-latency communication (delay of one-microsecond in communication).
This is 1,000 times faster - or 1/1000th the latency - than one millisecond throughput in compare to 5G.
Knowledge Base:
5G uses a new digital technology called Massive MIMO, which stands for multiple input multiple output, that
uses multiple targeted beams to spotlight and follow users around a cell site, improving coverage, speed and
capacity. Current network technologies operate like floodlights, illuminating an area but with lots of wastage
of the light/signal. Part of the roll-out of 5G involves installing Massive MIMO and 5G New Radio to all mobile
network base stations on top of the existing 4G infrastructure.
Millimeter wave spectrum: The 5G networks operate in the millimeter wave spectrum (30-300 GHz) which
have the advantage of sending large amounts of data at very high speeds because the frequency is so high, it
experiences little interference from surrounding signals.
Utility of 5G Applications: Combined with IoT, cloud, big data, AI, and edge computing, 5G could be a critical
enabler of the fourth industrial revolution.
Source: https://blog.forumias.com/answered-what-is-difference-between-5g-and-4g-technology-
discuss-importance-of-5g-technology-for-india/
https://indianexpress.com/article/technology/mobile-tabs/the-world-of-5g-bands-and-why-n78-is-
the-one-that-matters-7321657/
https://www.raconteur.net/technology/5g/4g-vs-5g-mobile-technology/
https://indianexpress.com/article/technology/tech-news-technology/6g-technology-launch-likely-by-
2023-end-or-2024-says-ashwini-vaishnaw-7638108/
https://whatis6g.com/difference-5g-6g/
https://www.cashify.in/explained-what-is-6g-technology-future-of-
telecom#:~:text=The%206G%20spectrum%20will%20land,deliver%20500mbps%20to%201gbps%20speed.
https://3g.co.uk/guides/3g-what-is-3g-explained-in-simple-terms

Q.27) What is “Virtual Private Network”?


a) It is a private compute network of an organization where he remote users can transmit encrypted
information through the server of the organization.
b) It is a computer network across a public internet that provides users access to their organization’s
network while maintaining the security of the information transmitted.
c) It is a computer network in which users can access a shared pool of computing resources through a service
provider
d) None of the statements (a), (b) and (c) given above is a correct description of Virtual Private Network.

Ans: b
Exp) Option b is correct.
Virtual private network, or VPN, is an encrypted connection over the Internet from a device to a network.
The encrypted connection helps ensure that sensitive data is safely transmitted. It prevents unauthorized
people from eavesdropping on the traffic and allows the user to conduct work remotely.
Source: UPSC 2011

Q.28) With reference to modern Communication technologies, consider the following statements:
1. Unlike a bar-code, Radio-frequency identification (RFID) tags need not be in line of sight to be read.
2. Unlike Near field communication (NFC) technology, RFID devices can act not only as a reader, but also as
a tag.
3. Bluetooth Low Energy (BLE) devices remain in sleep mode unless a connection initiates.
Which of the above options/statements is/are correct?
a) 1 and 2 only
b) 2 only
c) 1 and 3 only

Forum Learning Centre: Delhi - 2nd Floor, IAPL House, 19 Pusa Road, Karol Bagh, New Delhi - 110005 | Patna - 2nd floor, AG Palace, E Boring Canal Road,
Patna, Bihar 800001 | Hyderabad - 1st & 2nd Floor, SM Plaza, RTC X Rd, Indira Park Road, Jawahar Nagar, Hyderabad, Telangana 500020
9821711605 | https://academy.forumias.com | admissions@forumias.academy | helpdesk@forumias.academy
Page 15 of 29

SFG 2022 | LEVEL 1 | Test #37 – Solutions |


d) 1, 2 and 3

Ans) c
Exp) Option c is correct.
Near field communication (NFC), Radio-frequency identification (RFID) and Bluetooth Low Energy (BLE) are
modern communication technologies that allow one to automatically identify objects or people and due to
their different operating characteristics, complete each other.
Statement 1 is correct. RFID stands for ‘Radio-frequency identification’ where digital data is encoded in RFID
tags or smart labels which is captured by a reader. It is similar to barcoding where data from a tag is captured
by a device. The disadvantage with methods like a bar-code is that it needs to be in the line of sight to be
read whereas RFID tags need not be in line of sight.

Statement 2 is incorrect. Near field communication (NFC) is the set of protocols that enables electronic
devices to establish radio communication with each other by touching the devices together or bringing them
into proximity to a distance of typically 10cm or less. It works by the principle of induction.
NFC is also based on the RFID protocols. The main difference to RFID is that an NFC device can act not only
as a reader, but also as a tag (not the other way round). In peer-to-peer mode, it is also possible to transfer
information between two NFC devices.

Statement 3 is correct. Bluetooth Low Energy is for specific use cases with limited data transfer and
requires significantly less energy than classic Bluetooth—hence the name Low Energy. The hidden difference
is that Bluetooth Low Energy remains in sleep mode unless a connection initiates. The actual connection
times only last a few milliseconds, unlike Bluetooth, which connects for a few seconds or a few hours at a
time. These short connections are necessary because data rates are significantly higher (1 Mb per second). It
is radio technology that allows devices to be networked within a distance of about 10 meters.
BLE is not voice compatible.

Forum Learning Centre: Delhi - 2nd Floor, IAPL House, 19 Pusa Road, Karol Bagh, New Delhi - 110005 | Patna - 2nd floor, AG Palace, E Boring Canal Road,
Patna, Bihar 800001 | Hyderabad - 1st & 2nd Floor, SM Plaza, RTC X Rd, Indira Park Road, Jawahar Nagar, Hyderabad, Telangana 500020
9821711605 | https://academy.forumias.com | admissions@forumias.academy | helpdesk@forumias.academy
Page 16 of 29

SFG 2022 | LEVEL 1 | Test #37 – Solutions |


Knowledge Base: Components of RFID system
It consists of three components – an RFID tag or label, a reader and an antenna. The tag contains a circuit
and an antenna to transmit data to the reader. The reader in turn converts the waves which have been
transmitted to a usable form of data. The information is then transferred to a host computer system, where
data can be stored and analyzed.
Applications of RFID technology
1. Logistics and supply chain – With the visibility provided by RFID in the supply chain, It can help increase
efficiency, reduce errors and improve quality.
2. Inventory tracking – Tracking items through the supply chain to the point of scale can be done in a matter
of minutes with the help of a RFID reader.
3. Tracking attendees – Managing a conference with an RFID based solution will help eliminate the need for
long registration lines.
4. RFID based access – In certain areas an increased level of security is required. RFID control tags can
restrict access to only those people which have been pre-approved.
5. Real time location system – With this system, you can track the real time location of employees and assets.
It helps gauge the effectiveness of a floor-based plan as well as track the location of resources.
Source: https://www.ecom-ex.com/blog/post/rfid-vs-nfc-what-is-the-difference/
https://www.quicsolv.com/internet-of-things/difference-rfid-ble-nfc/
https://www.abr.com/what-is-rfid-how-does-rfid-work/
https://unikey.com/nfc-vs-rfid-vs-ble/
https://tapkey.com/blogs/smart-access-control/nfc-ble-rfid-difference
https://ytd2525.wordpress.com/2014/02/17/ble-vs-nfc-vs-rfid-learn-the-differences/
https://www.link-labs.com/blog/bluetooth-vs-bluetooth-low-energy
https://blog.roambee.com/supply-chain-technology/evolution-in-supply-chain-visibility-barcodes-to-
rfid-to-ble-beacons

Q.29) With reference to communications technologies, what is/are the differences between RADAR and
LiDAR?
1. Unlike LiDAR, RADAR can easily operate in night and cloudy weather conditions.
2. To detect objects, LiDAR uses comparatively long wavelength than RADAR.
Which of the above statements is/are correct?
a) 1 only
b) 2 only
c) Both 1 and 2
d) Neither 1 nor 2

Ans) a
Exp) Option a is correct.
Essentially, RADAR (Radio Detection And Ranging) and LiDAR (Light Detection And Ranging) work towards
the same goal, that is to detect objects. But these technologies do operate and provide results in notably
different ways when compared to each other. At the basic level, RADAR and LiDAR differ because they don’t
use the same wavelength type.
LiDAR stands for Light Detection and Ranging. LiDAR is a remote sensing method that uses light in the form
of a pulsed laser to measure ranges (variable distances) to the Earth.
The RADAR system works in much the same way as LiDAR, with the big difference that it uses radio waves
instead of laser or LED light. It transmits radio waves from a rotating or fixed antenna and measure the time
of flight of the reflected signal.
Statement 1 is correct. RADAR can detect objects at long distance and through fog or clouds. But its lateral
resolution is limited by the size of the antenna. Unlike RADAR technology, LiDAR pulses are adversely
affected by atmospheric weather conditions such as dense fogs, smoke, and even rain. The light pulses will
be distorted during flight and this will affect the accuracy of the data collected.
Statement 2 is incorrect. The difference between LiDAR and RADAR is their wavelength. LIDAR uses short
wavelength as it is based on light to detect small objects. While Radar uses radio waves of longer wavelength
and hence does not allow the detection of small objects. RADAR cannot provide the user with the precise
image of an object because of the longer wavelength. The wavelength of RADAR is between 30 cm and 3 mm,
while LiDAR has a micrometer range wavelength.
Source: https://cprimestudios.com/blog/lidar-vs-radar-pros-and-cons-autonomous-driving
https://flyguys.com/lidar-vs-radar/
https://www.yellowscan-lidar.com/knowledge/lidar-vs-radar/

Forum Learning Centre: Delhi - 2nd Floor, IAPL House, 19 Pusa Road, Karol Bagh, New Delhi - 110005 | Patna - 2nd floor, AG Palace, E Boring Canal Road,
Patna, Bihar 800001 | Hyderabad - 1st & 2nd Floor, SM Plaza, RTC X Rd, Indira Park Road, Jawahar Nagar, Hyderabad, Telangana 500020
9821711605 | https://academy.forumias.com | admissions@forumias.academy | helpdesk@forumias.academy
Page 17 of 29

SFG 2022 | LEVEL 1 | Test #37 – Solutions |


Q.30) In the context of digital technologies for entertainment, consider the following statements:
1. In Augmented Reality (AR), a simulated environment is created and the physical world is completely shut
out.
2. In Virtual Reality (VR), images generated from a computer are projected onto real-life objects or
surroundings.
3. AR allows individuals to be present in the world and improves the experience using the camera of smart-
phone or PC.
4. VR closes the world, and transposes an individual, providing complete immersion experience.
Which of the statements given above is/are correct?
a) 1 and 2 only
b) 3 and 4
c) 1, 2 and 3
d) 4 only

Ans: b
Exp) Option b is correct.
 Augmented Reality (AR) is an interactive experience of a real-world environment where designers
enhance parts of users’ physical world with computer-generated input. A Simple example is, during
football games, broadcasters use AR to draw lines on the field to illustrate and analyse plays. AR allows
individuals to be present in the world and improves the experience using the camera of smart-phone or
PC.
 Virtual Reality (VR) is a computer-generated environment with scenes and objects that appear to be real,
making the user feel they are immersed in their surroundings. Virtual Reality places the user inside an
experience.
Source: UPSC 2019

Q.31) With reference to computed tomography (CT) scan and X-RAYS, consider the following statements:
1. Unlike chest X-ray, Chest CT scan delivers 3-dimension view of the chest.
2. CT scan uses more ionizing radiation than X-ray.
Which of the above statements is/are correct?
a) 1 only
b) 2 only
c) Both 1 and 2
d) Neither 1 nor 2

Ans) c
Exp) Option c is correct.
A computerized tomography (CT) scan combines a series of X-ray images taken from different angles around
the body. It uses computer processing to create cross-sectional images (slices) of the bones, blood vessels
and soft tissues inside your body. CT scan images provide more-detailed information than plain X-rays do.
Statement 1 is correct. Chest CT scan is a more detailed type of chest X-ray that compounds the power of
X-rays and computers to deliver a 3D view of your chest. A CT scan makes a number of detailed slices
(pictures), which then are merged into one projection. This creates a clear view that shows the position,
shape, and size of the organs in your chest.
A CT scan produces detailed images of organs, bones, soft tissue and blood vessels and can be used to more
easily diagnose cancer, heart disease, appendicitis, musculoskeletal disorders, trauma and infectious
diseases.
A chest X-ray helps to indicate abnormal formations or a large variety of chest diseases such as pneumonia,
cystic fibrosis, emphysema, cancer, etc. In addition, a chest X-ray is often used for an emergency diagnosis
due to its fast and easy usage. It provides a 2-dimensional view of the chest.
Statement 2 is correct. Computed tomography (CT scan) contains 5 to 10 times more ionizing radiation
than X-ray. CT scans are an important component of Covid diagnosis as well as determining the severity of
the disease. CT scan is far more sensitive than saturation monitoring by pulse oximetry in detecting lung
damage, and can help timely administration of steroids.
Source: https://floridachest.com/pulmonary-blog/chest-x-ray-chest-ct-scan-differences
https://www.northcentralsurgical.com/blog/whats-the-difference-between-an-x-ray-ct-scan-and-mri-
140.html
https://theprint.in/health/ct-scan-is-like-5-10-x-rays-not-300-400-docs-body-refutes-gulerias-
unscientific-comments/653033/

Forum Learning Centre: Delhi - 2nd Floor, IAPL House, 19 Pusa Road, Karol Bagh, New Delhi - 110005 | Patna - 2nd floor, AG Palace, E Boring Canal Road,
Patna, Bihar 800001 | Hyderabad - 1st & 2nd Floor, SM Plaza, RTC X Rd, Indira Park Road, Jawahar Nagar, Hyderabad, Telangana 500020
9821711605 | https://academy.forumias.com | admissions@forumias.academy | helpdesk@forumias.academy
Page 18 of 29

SFG 2022 | LEVEL 1 | Test #37 – Solutions |


Q.32) Which of the following are the methods used for Waste to energy conversion?
1. Torrefaction
2. Polycrack
3. Soil Liquefaction
4. Pulverization
Select the correct answer using the code given below.
(a) 1 and 2 only
(b) 2, 3 and 4 only
(c) 4 only
(d) 1 and 3 only

Ans) a
Exp) Option a is correct.
Energy can be generated in form of electricity and/or heat from the primary treatment of waste. It is done
through Waste-to-energy process.
Waste-to-Energy (WtE) or Energy-from-Waste (EfW) is a form of energy recovery and the process of
generating energy in the form of electricity and/or heat by processing of waste into a fuel source. There are
various kind of Waste to energy processes as follows:
Statement 1 is correct. The torrefaction technology involves heating straw, grass, sawmill residue, and wood
biomass to 250 degrees Celsius – 350 degrees Celsius. This changes the elements of the biomass into ‘coal-
like’ pellets. These pellets can be used for combustion along with coal for industrial applications like steel and
cement production.

Statement 2 is correct. Polycrack technology is the world’s first patented heterogeneous catalytic process
that converts multiple feedstocks into hydrocarbon liquid fuels, gas, carbon, and water.
The process is a closed-loop system and does not emit any hazardous pollutants into the atmosphere.
The combustible, non-condensed gases are re-used for providing energy to the entire system and thus, the
only emission comes from the combustion of gaseous fuels.
This process will produce energy in the form of light diesel oil which is used to light furnaces.
Indian Railways has commissioned country’s first governmental Waste to Energy Plant based on polycrack
technology, at Bhubaneswar in East Coast Railway.
Statement 3 is incorrect. Liquefaction is a phenomenon in which the strength and stiffness of a soil is
reduced by earthquake shaking or other rapid loading. Liquefaction and related phenomena have been
responsible for tremendous amounts of damage in historical earthquakes around the world. It is not a waste
to energy conversion technique.
Statement 4 is incorrect. Pulverization (comminution, crushing, grinding) is the process of applying an
external force to a (solid) material of a certain size to destroy it and reduce it into pieces that are smaller
than the original size. In this method, collected solid waste is powdered by grinding machines, thereby
changing its volume and physical characteristics. This pulverized solid waste is further disposed-off by land
filling. This is costly technique. It is not a waste to energy conversion technique.
Knowledge Base:
India is testing torrefaction that can convert rice stubble into ‘bio-coal’. Torrefaction is a thermochemical
process typically at 200-350 °C in the absence of oxygen, at atmopsheric pressure with low particle heating
rates and a reactor time of one hour. The process causes biomass to partly decompose, creating torrefied
biomass or char, also referred to as 'biocoal'. Biocoal has a higher energy content per unit volume, and
torrefaction followed by pelletisation at the harvest sites facilitates transport over longer distances.
The government has funded a pilot project at the National Agri-Food Biotechnology Institute in Mohali
(Punjab) with a Swedish company to evaluate the feasibility of the technology.
Source: https://pib.gov.in/PressReleasePage.aspx?PRID=1600784

Forum Learning Centre: Delhi - 2nd Floor, IAPL House, 19 Pusa Road, Karol Bagh, New Delhi - 110005 | Patna - 2nd floor, AG Palace, E Boring Canal Road,
Patna, Bihar 800001 | Hyderabad - 1st & 2nd Floor, SM Plaza, RTC X Rd, Indira Park Road, Jawahar Nagar, Hyderabad, Telangana 500020
9821711605 | https://academy.forumias.com | admissions@forumias.academy | helpdesk@forumias.academy
Page 19 of 29

SFG 2022 | LEVEL 1 | Test #37 – Solutions |


https://www.thehindu.com/news/cities/Delhi/india-tests-swedish-technology-to-reduce-stubble-
burning/article30153719.ece
https://depts.washington.edu/liquefy/html/what/what1.html
https://www.thinkymixer.com/en-us/library/glossary/pulverization-1-definition-and-purpose/

Q.33) Consider the following statements about a digital signature:


1. an electronic record that identifies the certifying authority issuing it
2. used to serve as a proof of identity of an individual to access information or server on Internet
3. an electronic method of signing an electronic document and ensuring that the original content is
unchanged
Which of the statements given above is/are correct?
a) 1 only
b) 2 and 3 only
c) 3 only
d) 1, 2 and 3

Ans: c
Exp) Option c is correct.
 The statements 1 and 2 are correct for “Digital Signature Certificates” but not for “Digital Signatures”.
 A digital signature is an electronic, encrypted, stamp of authentication on digital information such as
email messages, macros, or electronic documents. A signature confirms that the information originated
from the signer and has not been altered.
 The Central Government has appointed a Controller of Certifying Authorities who grants a license to the
Certifying Authorities to issue digital signature certificates to the subscriber.
Source:UPSC 2019

Q.34) Which of the following statements is correct about Hyperloop?


a) It is a high speed grounded or elevated transportation system that uses magnets to elevate trains and
propel them forward.
b) It is a high speed highly computerized grounded transportation system with in-cab signaling and power
units.
c) It is a high-speed ground transportation with contactless propulsion system using low air pressure.
d) It is a rail-less transportation system that runs on electricity following virtual routes.

Ans) c
Exp) Option c is correct.
Statement a is incorrect. Maglev short for magnetic levitation is a high speed grounded or elevated
transportation system which uses magnets to elevate trains and propel them forward at a high speed, In
Maglev, superconducting magnets suspend a train car above a U-shaped concrete guideway. Like ordinary
magnets, these magnets repel one another when matching poles face each other.
Statement b is incorrect. The TGV (Train à Grande Vitesse) is the French high speed grounded
transportation system in France. TGV trainsets are essentially symmetric and reversible, with a locomotive,
also called power unit or power car, coupled at each end. The system uses exclusive in-cab signalling for
high-speed running. TGV lines do not have lineside signals; they are too difficult to read at speed. All
signalling information is transmitted to the train through the rails, and appears to the engineer in the cab. In
general, TGV trainsets are heavily computerized, and many important functions are controlled digitally.
Statement c is correct. Hyperloop is an ultra-high-speed ground transportation system for passenger and
cargo proposed as a concept by Elon Musk, CEO of Tesla and SpaceX, in 2013. It consists of partially
evacuated tubes and passenger or cargo pods moving at very high speeds. The Hyperloop system connects
mobility hubs in large metropolitan areas, and pressurized vehicles, usually called pods, which can move
at very high speeds with contactless levitation and propulsion systems as well as to the low aerodynamic
drag.
Statement d is incorrect. The Autonomous Rail Rapid Transit(ART) system is a rail-less train system designed to
be used for urban passenger transport which can carry up to 300 passengers across three carriages at speeds of up
to 70 kilometres per hour. ART bears the physical appearance of a train but it doesn’t rely on following a track. Instead, it
follows a virtual route using an electric powertrain and tires. This vehicle is emission free, as it runs on electricity.
Source: http://www.railfaneurope.net/tgv/background.html
https://about.bgov.com/news/hyperloop-nyc-maglev-train-line-up-for-infrastructure-money/
https://www.energy.gov/articles/how-maglev-works
https://tumhyperloop.de/about-hyperloop/#hyperloop-faq
https://www.nbcnews.com/mach/science/new-type-electric-transport-being-tested-china-ncna816401

Forum Learning Centre: Delhi - 2nd Floor, IAPL House, 19 Pusa Road, Karol Bagh, New Delhi - 110005 | Patna - 2nd floor, AG Palace, E Boring Canal Road,
Patna, Bihar 800001 | Hyderabad - 1st & 2nd Floor, SM Plaza, RTC X Rd, Indira Park Road, Jawahar Nagar, Hyderabad, Telangana 500020
9821711605 | https://academy.forumias.com | admissions@forumias.academy | helpdesk@forumias.academy
Page 20 of 29

SFG 2022 | LEVEL 1 | Test #37 – Solutions |


Q.35) With reference to Defense System of India, consider the following statements:
1. Ballistic missiles are powered initially by a rocket or series of rockets in stages.
2. Unlike Cruise missiles, ballistic missiles can carry nuclear warheads.
3. S-400 is an anti-missile defence system which can engage both cruise missiles and ballistic missiles.
Which of the statements above is/are correct?
a) 1 and 2 only
b) 2 and 3 only
c) 1 and 3 only
d) 1, 2 and 3

Ans) c
Exp) Option c is correct.
The Integrated Guided Missile Development Programme (IGMDP) conceived by renowned scientist Dr. A P J
Abdul Kalam to enable India attain self-sufficiency was started in 1983 completed in March 2012.It was an
Indian Ministry of Defence programme for the research and development of the comprehensive range of
missiles. India now moved on with the new Indian Ballistic Missile Defence Program as an initiative to develop
and deploy a multi-layered ballistic missile defence system to protect India from ballistic missile attacks.
Statement 1 is correct. Ballistic missiles are powered by rockets initially but then they follow an
unpowered, free-falling trajectory towards their target. While cruise missiles are self-propelled systems till
the end of its flight.
Statement 2 is incorrect. Both cruise missiles and Ballistic missiles can carry either nuclear or conventional
warheads.
Statement 3 is correct. The S-400 Triumph air defence system developed by Almaz Central Design Bureau
of Russia integrates a multifunction radar, autonomous detection and targeting systems, anti-aircraft missile
systems, launchers, and command and control centre. The S-400 missile defence system is equipped with
four different missiles which can engage enemy aircraft, ballistic missiles, cruise missiles and AWACS planes
at 400 km, 250 km, the medium-range 120 km and the short-range 40 km.
Source: https://economictimes.indiatimes.com/news/defence/india-deploys-first-s-400-air-defence-
system-in-punjab-sector-to-take-care-of-aerial-threats-from-both-china-
pakistan/articleshow/88397852.cms
https://www.indiatimes.com/news/india/india-s-400-surface-to-air-missile-defense-system-
557308.html?picid=2217121
https://armscontrolcenter.org/wp-content/uploads/2017/04/Ballistic-vs.-Cruise-Missiles-Fact-
Sheet.pdf
https://www.defencexp.com/full-list-of-indian-air-defence-systems/

Q.36) With reference to communication technologies, what is/are the difference/ differences between LTE
(Long-Term Evolution) and VoLTE (Voice over Long-Term Evolution)?
1. LTE is commonly marketed as 3G and VoLTE is commonly marketed as advanced 3G.
2. LTE is data-only technology and VoLTE is voice-only technology.
Select the correct answer using the code given below.
a) 1 only
b) 2 only
c) Both 1 and 2
d) Neither 1 nor 2

Ans: d
Exp) Option d is correct.
 LTE is commonly marketed as 4G, it supports download speeds of upto 100 mbps and upload speeds upto
50 mbps.
 VoLTE can simultaneously send data and voice over the network without reducing quality of voice call.
Source: UPSC 2019

Q.37) With reference to Polymerase chain reaction (PCR), consider the following statements:
1. It is a laboratory technique used to amplify DNA sequences.
2. It is a highly sensitive technique with the potential to produce millions to billions of copies of a specific
product.
3. It has the ability to test for multi-drug resistance against microbes or viruses.
Which of the statements above is/are correct?
a) 1 and 2 only
b) 2 and 3 only
c) 1 and 3 only

Forum Learning Centre: Delhi - 2nd Floor, IAPL House, 19 Pusa Road, Karol Bagh, New Delhi - 110005 | Patna - 2nd floor, AG Palace, E Boring Canal Road,
Patna, Bihar 800001 | Hyderabad - 1st & 2nd Floor, SM Plaza, RTC X Rd, Indira Park Road, Jawahar Nagar, Hyderabad, Telangana 500020
9821711605 | https://academy.forumias.com | admissions@forumias.academy | helpdesk@forumias.academy
Page 21 of 29

SFG 2022 | LEVEL 1 | Test #37 – Solutions |


d) 1, 2 and 3

Ans) d
Exp) Option d is correct.
The polymerase chain reaction (PCR) was first discovered by Kary Mullis in 1990 which radically transformed
biological science. For the first time, it allowed for specific detection and production of large amounts of
DNA.
Statement 1 is correct. Polymerase chain reaction (PCR) is a laboratory technique used to amplify DNA
sequences. The method involves using short DNA sequences called primers to select the portion of the
genome to be amplified. While, DNA sequencing is a laboratory technique used to determine the exact
sequence of bases (A, C, G, and T) in a DNA molecule.
Statement 2 is correct. It is a highly sensitive technique with the potential to produce millions to billions of
copies of a specific product for sequencing, cloning, and analysis. Because PCR is a highly sensitive technique,
any form of contamination of the sample by even trace amounts of DNA can produce misleading results
Statement 3 is correct. The one of the most important benefits of PCR is the ability to test for multi-drug
resistance. The strength of multiplex PCR is the ability to test for multiple antibiotic genes with one sample,
guiding specific antimicrobial treatment while reducing the chance of developing drug resistance. Another
way PCR can evaluate sensitivity to antimicrobials is by quantitative analysis of viral load after treatment.
Source: https://www.genome.gov/genetics-glossary/DNA-Sequencing
https://www.genome.gov/genetics-glossary/Polymerase-Chain-Reaction
https://www.ncbi.nlm.nih.gov/pmc/articles/PMC6778471/
https://www.ncbi.nlm.nih.gov/pmc/articles/PMC4102308/

Q.38) Which of the following statements correctly defines Chemosynthesis?


a) It is the process by which certain microbes create energy by mediating chemical reactions.
b) It is defined as directional cell movement of cells towards concentration gradients of solubilized
attractants.
c) It is defined as random cell movement in the absence of chemoattractant gradients.
d) It is a drug treatment that uses powerful chemicals to kill fast-growing cells in your body.

Ans) a
Exp) Option a is correct.
Most life on Earth is dependent upon photosynthesis, the process by which plants make energy from
sunlight. However, at hydrothermal vents in the deep ocean a unique ecosystem has evolved in the absence
of sunlight, and its source of energy is completely different: chemosynthesis.
Statement a is correct. Chemosynthesis is the process by which certain microbes create energy by mediating
chemical reactions. The chemosynthetic microbes provide the foundation for biological colonization of vents.
Chemosynthetic microbes live on or below the seafloor, and even within the bodies of other vent animals as
symbionts.
Statement b is incorrect. Chemotaxis is defined as directional cell movement of cells towards concentration
gradients of solubilized attractants. Chemotaxis is a fundamental form of cell behaviour that involves a
complex response of a cell to an external stimulus
Statement c is incorrect. Chemokinesis is defined as random cell movement in the absence of
chemoattractant gradients, chemotactic agents that induce a cell to migrate toward them.
Statement d is incorrect. Chemotherapy is a drug treatment that uses powerful chemicals to kill fast-
growing cells in your body. Chemotherapy is most often used to treat cancer, since cancer cells grow and
multiply much more quickly than most cells in the body.
Source: https://www.pmel.noaa.gov/eoi/nemo/explorer/concepts/chemosynthesis.html
https://pubmed.ncbi.nlm.nih.gov/15274332/
https://www.sciencedirect.com/topics/medicine-and-dentistry/chemokinesis
https://www.mayoclinic.org/tests-procedures/chemotherapy/about/pac-20385033

Q.39) With reference to Visible Light Communication (VLC) technology, which of the following statements
are correct?
1. VLC uses electromagnetic spectrum wavelengths 375 to 780 nm.
2. VLC is known as long-range optical wireless communication.
3. VLC can transmit large amounts of data faster than Bluetooth.
4. VLC has no electromagnetic interference.
Select the correct answer using the code given below:
a) 1, 2 and 3 only
b) 1, 2 and 4 only
c) 1, 3 and 4 only

Forum Learning Centre: Delhi - 2nd Floor, IAPL House, 19 Pusa Road, Karol Bagh, New Delhi - 110005 | Patna - 2nd floor, AG Palace, E Boring Canal Road,
Patna, Bihar 800001 | Hyderabad - 1st & 2nd Floor, SM Plaza, RTC X Rd, Indira Park Road, Jawahar Nagar, Hyderabad, Telangana 500020
9821711605 | https://academy.forumias.com | admissions@forumias.academy | helpdesk@forumias.academy
Page 22 of 29

SFG 2022 | LEVEL 1 | Test #37 – Solutions |


d) 2, 3 and 4 only

Ans: c
Exp) Option c is correct.
Statement 1 is correct: Visible Light Communication (VLC) systems employ visible light for
communication that occupy the spectrum from 380 nm to 750 nm corresponding to a frequency spectrum
of 430 THz to 790 THz.
Statement 2 is incorrect: VLC is a communication technology in which the visible spectrum is modulated
to transmit data. Due to the limited propagation distance of the light emitting diodes (LEDs), VLC is a
short-range (and not long-range) wireless communication technology.
Statement 3 is correct: Aside from the massive size of the visible light spectrum, in comparison to entire
radio spectrum, light travels 300,000 Km per second, which is way faster than the 344 meters per second
traveled by radio waves in air. This means that communication using light is virtually instantaneous, which
also makes VLC the fastest means of communication among those commercially available in the market.
Thus the speed and size of data transfer is faster than radio frequency based Bluetooth technology.
Statement 4 is correct: Unlike radio frequency (RF) communication systems, VLC systems operate at
optical frequencies and emit no electromagnetic interference. Therefore, they can be safely deployed in
hospitals and healthcare facilities without any concerns on possible interference to medical instruments.
Source:UPSC 2020
https://www.sciencedirect.com/science/article/pii/S2352864816300335
https://www.researchgate.net/publication/279321580_Visible_Light_Communications_VLC_Technolog
y
https://lifi.co/visible-light-communication/
https://www.hyperiontechs.com/vlc/applications-of-vlc/

Q.40) Consider the following systems:


1. Cognitive computing can simulate human thought.
2. The personal digital assistants we have on our phones and computers are the highest example of cognitive
computing systems.
Which of the statements given above is/are correct?
a) 1 only
b) 2 only
c) Both 1 and 2
d) Neither 1 nor 2

Ans) a
Exp) Option a is correct.
Emerging technologies are science-based innovations with the potential to create a new industry or
transform an existing one which will substantially alter the business and social environment along with
emerging new cognitive computing models.
Cognitive computing, a subfield of artificial intelligence, simulates human thought processes in machines
using self-learning algorithms through data mining, pattern recognition, and natural language processing.
Statement 1 is correct. The goal of cognitive computing is to simulate human thought processes in a
computerized model. Using self-learning algorithms that use data mining, pattern recognition and natural
language processing, the computer can mimic the way the human brain works.
Statement 2 is incorrect. The personal digital assistants we have on our phones and computers now (Siri
and Google among others) are not true cognitive systems; they have a pre-programmed set of responses
and can only respond to a preset number of requests.
Source: https://www.forbes.com/sites/bernardmarr/2016/03/23/what-everyone-should-know-about-
cognitive-computing/?sh=33d90d415088
https://egyankosh.ac.in/bitstream/123456789/72081/1/Block-1.pdf
https://www.networkworld.com/article/3243111/what-is-fog-computing-connecting-the-cloud-to-
things.html
https://www.sciencedirect.com/journal/international-journal-of-cognitive-computing-in-
engineering#:~:text=will%20be%20considered.-
,Cognitive%20computing%20is%20the%20creation%20of%20self%2Dlearning%20systems%20that,%2C%2
0combining%20ideas%2C%20principles%20and%20%E2%80%A6

Q.41) Consider the following statements with reference to the applications of deoxyribonucleic acid (DNA)
computing:
1. DNA based computing uses biological molecules instead of silicon chips for computing.
2. DNA computing can be used in cryptography to build an intrusion detection model.

Forum Learning Centre: Delhi - 2nd Floor, IAPL House, 19 Pusa Road, Karol Bagh, New Delhi - 110005 | Patna - 2nd floor, AG Palace, E Boring Canal Road,
Patna, Bihar 800001 | Hyderabad - 1st & 2nd Floor, SM Plaza, RTC X Rd, Indira Park Road, Jawahar Nagar, Hyderabad, Telangana 500020
9821711605 | https://academy.forumias.com | admissions@forumias.academy | helpdesk@forumias.academy
Page 23 of 29

SFG 2022 | LEVEL 1 | Test #37 – Solutions |


3. Unlike traditional binary coding, DNA computing uses chemical bases to hide the messages.
4. DNA computing-based clustering involves using strands to assign edges and vertices.
Which of the statements given above is/are correct?
a) 1, 2 and 3 only
b) 2, 3 and 4 only
c) 1, 2 and 4 only
d) 1, 2, 3 and 4

Ans) d
Exp) Option d is correct.
DNA computing is a branch of biomolecular computing concerned with the use of DNA as a carrier of
information to make arithmetic and logic operations.
Statement 1 is correct. DNA based computing uses biological molecule instead of silicon chip for
computing. So (A)(T)(G)(C) will be used Instead of 0 and 1used in computer processing. In the field of DNA-
based computing, the DNA contains the information, but the molecules are floating around in solution.
Statement 2 is correct. DNA computing can be used in cryptography. Deploying DNA algorithms in
cryptography to build an intrusion detection model is the most recent development of DNA computing
model. The ability to store 108 terabytes of data in 1 gram of DNA has led to the potential holding a huge one-
time pad.
Statement 3 is correct. Another application of DNA computing is in DNA steganography, in which a novel
method was used to hide the messages in a microdot. Instead of the traditional binary encoding, each letter
was denoted by three chemical bases i.e., the letter A was encoded by CGA. These messages are then encoded
into DNA sequences and concealed by mixing it in a tube with a large amount of sonicated random human
DNA. This led to the formation of microdots, which was then decoded by the receiver with appropriate
primers (short sequence with complementary bases).
Statement 4 is correct. Clustering deals with deriving highly meaningful relationships in a complex collection
of data by creating a structure using various concepts and algorithms. DNA- based clustering involves using
strands to assign edges and vertices. Iterative calculations are performed for every produced cluster to
improve quality. This method is of particular interest when dealing with large heterogeneous data with an
unknown number of clusters. It helps in reducing the time complexity by high parallelism features of DNA.
Source: https://www.section.io/engineering-education/dna-computing/

Q.42) To meet its rapidly growing, energy demand, some opine that India should pursue research and
development on thorium as the future fuel of nuclear energy. In this context, what advantage does thorium
hold over uranium?
1. Thorium is far more abundant in nature than uranium.
2. On the basis of per unit mass of mined mineral, thorium can generate more energy compared to natural
uranium.
3. Thorium produces less harmful waste compared to uranium.
Which of the statements given above is/are correct?
a) 1 only
b) 2 and 3 only
c) 1 and 3
d) 1, 2 and 3

Ans: d
Exp) Option d is correct.
Thorium (Th) is radioactive chemical element of the actinoid series of the periodic table. Thorium abundance
in the earth’s continental crust is generally estimated to be three to five times that of uranium
It is estimated that one ton of thorium can produce as much energy as 35 tons of uranium in a liquid
fluoride thorium reactor. Thorium produces less harmful waste compared to uranium.
All of the above are advantages of Thorium over Uranium.
Source: UPSC [2012]

Q.43) Consider the following statements:


1. Innate immunity offers a lifelong protection with fast acting response from the immune system.
2. Adaptive immunity is an organism’s acquired immunity to a specific pathogen.
3. Helper T cells are the most important cells in innate immunity.
Which of the statements given above is/are correct?
a) 1 and 2 only
b) 2 and 3 only
c) 1 and 3 only

Forum Learning Centre: Delhi - 2nd Floor, IAPL House, 19 Pusa Road, Karol Bagh, New Delhi - 110005 | Patna - 2nd floor, AG Palace, E Boring Canal Road,
Patna, Bihar 800001 | Hyderabad - 1st & 2nd Floor, SM Plaza, RTC X Rd, Indira Park Road, Jawahar Nagar, Hyderabad, Telangana 500020
9821711605 | https://academy.forumias.com | admissions@forumias.academy | helpdesk@forumias.academy
Page 24 of 29

SFG 2022 | LEVEL 1 | Test #37 – Solutions |


d) 1, 2 and 3

Ans) a
Exp) Option a is correct.
The immune system fights germs and foreign substances on the skin, in the tissues of the body and in bodily
fluids such as blood. The immune system is made up of two parts: the innate, (general) immune system and
the adaptive (specialized) immune system. These two systems work closely together and take on different
tasks.
Statement 1 is correct. Innate immunity is also known as genetic or natural immunity. Innate immunity is
the immunity that one is born with. This type of immunity is written in one’s genes, offering lifelong
protection. The innate immune response is fast acting and non-specific, meaning it does not respond
differently based on the specific virus or bacteria that it detects.
Statement 2 is correct. Adaptive immunity is an organism’s acquired immunity to a specific pathogen. It
is also referred to as acquired immunity. Adaptive immunity is not immediate, nor does it always last
throughout an organism’s entire lifespan, although it can. The adaptive immune response is marked by clonal
expansion of T and B lymphocytes, releasing many antibody copies to neutralize or destroy their target
antigen.
Statement 3 is incorrect. Helper T cells are the most important cells in adaptive immunity, as they are
required for almost all adaptive immune responses. They not only help activate B cells to secrete antibodies
and macrophages to destroy ingested microbes, but they also help activate cytotoxic T cells to kill infected
target cells. As dramatically demonstrated in AIDS patients, without helper T cells we cannot defend
ourselves even against many microbes that are normally harmless. Helper T cells can only function when
activated to become effector cells.
Knowledge base: The innate immune system encompasses physical barriers and chemical and cellular
defenses.
 Physical barriers protect the body from invasion. These include things like the skin and eyelashes.
 Chemical barriers are defense mechanisms that can destroy harmful agent. Examples include tears,
mucous, and stomach acid.
 Cellular defenses of the innate immune response are non-specific. These cellular defenses identify
pathogens and substances that are potentially dangerous and take steps to neutralize or destroy them.
Source: https://www.technologynetworks.com/immunology/articles/innate-vs-adaptive-immunity-
335116
https://www.ncbi.nlm.nih.gov/books/NBK26827/#:~:text=Helper%20T%20cells%20are%20arguably,to%
20kill%20infected%20target%20cells.

Q.44) Which of the following is/are the applications of Nanotechnology:


1. Green Technology
2. Cosmetics
3. Construction of spacecraft and satellites
4. New forms of carbon
5. To create artificial organs and implants
Select the correct answer using the codes given below:
a) 1, 2, and 3 only
b) 2, 3, 4 and 5 only
c) 1, 2, 3 and 5 only
d) 1, 2, 3, 4 and 5

Ans) d
Exp) Option d is correct.
Nanotechnology applications in development can be broadly divided into several thematic areas:
the development of the tools that enable the research and ultimately the technology;
 applications relating to new or improved materials; applications within the sphere of electronics and IT;
 advances in health and medicine;
 improvements in cosmetic products and advances in food technology;
 developments in products for military and security use, and space exploration; and Products and
processes to improve the environment.
Nano technology has tremendous development in the fields of material science, electronics, biomedical
science, biotechnology, Military and the environment.
Statement 1 is correct. Nanotechnology can be used to prevent, monitor and alleviate a wide range of
environmental problems, while significantly reducing cost and improving performance. Current and future
applications of nanotechnology will allow us to:
 Develop new "green" processing technologies that minimize the amount of undesired byproducts;

Forum Learning Centre: Delhi - 2nd Floor, IAPL House, 19 Pusa Road, Karol Bagh, New Delhi - 110005 | Patna - 2nd floor, AG Palace, E Boring Canal Road,
Patna, Bihar 800001 | Hyderabad - 1st & 2nd Floor, SM Plaza, RTC X Rd, Indira Park Road, Jawahar Nagar, Hyderabad, Telangana 500020
9821711605 | https://academy.forumias.com | admissions@forumias.academy | helpdesk@forumias.academy
Page 25 of 29

SFG 2022 | LEVEL 1 | Test #37 – Solutions |


 Detect and remove the finest contaminants from air, water, and soil, which would enhance the ability
of governments to respond to terrorist threats and ensure the safety of water supplies.
Statement 2 is correct. Cosmetics and personal products companies have been extremely active in using
nanotechnology to improve their existing products and to develop new ones.
Statement 3 is correct. In the field of military, improved materials, lighter but with tough, heat resistant
properties, are being used in the design and construction of spacecraft and satellites, and this process will
gain from nanotechnology. There is also the possibility of nanotechnology facilitating improvements in
civilian security equipment.
Statement 4 is correct. The science of metals, ceramics, colloids and polymers, has always concerned itself
with controlling the structure of materials on the Nano scale. Some specific areas in which Nano science
technology is contributing to materials science now include: new forms of carbon; Nano composites;
quantum dots and wires; and nanostructured materials produced by self-assembly.
Statement 5 is correct. The medical area of Nano science application is projected as one of the most
potentially valuable, with many projected benefits to humanity. With the advent of new materials, and the
synergy of nanotechnologies and biotechnologies, it could be possible to create artificial organs and
implants that are more akin to the original, through cell growth on artificial scaffolds or biosynthetic coatings
that increase biocompatibility and reduce rejection.
Source: https://egyankosh.ac.in/bitstream/123456789/35558/1/Unit-3.pdf

Q.45) With reference to recent developments regarding 'Recombinant Vector Vaccines', consider the
following statements:
1. Genetic engineering is applied in the development of these vaccines.
2. Bacteria and viruses are used as vectors.
Which of the statements given above is/are correct?
a) 1 only
b) 2 only
c) Both 1 and 2
d) Neither 1 nor 2

Ans) c
Exp) Option c is correct.
Statement 1 is correct. Genetic engineering is applied in the development of Recombinant vector vaccines.
Genetic engineering involves techniques to alter the chemistry of genetic material (DNA and RNA), to
introduce these into host organisms and thus change the phenotype of the host organism. The techniques
of genetic engineering include creation of recombinant DNA, use of gene cloning and gene transfer.
Statement 2 is correct. Recombinant vaccines are made of a viral or bacterial vector. They are used as
channels for entry into host organisms.
Knowledge Base:
Viral vector-based vaccines use the body’s own cells to produce them. They do this by using a modified virus
(the vector) to deliver genetic code for antigen, in the case of COVID-19 spike proteins found on the surface
of the virus, into human cells. Gene engineering is applied in the development of recombinant vector
vaccines. The use of recombinant proteins allows the targeting of immune responses focused against few
protective antigens.
Source: https://www.ncbi.nlm.nih.gov/pmc/articles/PMC3854212/
https://www.gavi.org/vaccineswork/what-are-viral-vector-based-vaccines-and-how-could-they-be-
used-against-covid-19
https://www.sciencedirect.com/topics/medicine-and-dentistry/recombinant-vaccine
https://ncert.nic.in/ncerts/l/lebo111.pdf

Q.46) Consider the following statements with reference to the Big Data:
1. Big Data refers to a large amount of data which is of structured, semi-structured or unstructured nature.
2. Big Data problems can be solved by Cloud computing technology.
3. NITI Aayog is planning to develop the ‘National Data & Analytics Platform’, for evidence-based policymaking
guided by Big Data.
Which of the statements given above is/are correct?
a) 1 and 2 only
b) 2 and 3 only
c) 1 and 3 only
d) 1, 2 and 3

Ans) d
Exp) Option d is correct.

Forum Learning Centre: Delhi - 2nd Floor, IAPL House, 19 Pusa Road, Karol Bagh, New Delhi - 110005 | Patna - 2nd floor, AG Palace, E Boring Canal Road,
Patna, Bihar 800001 | Hyderabad - 1st & 2nd Floor, SM Plaza, RTC X Rd, Indira Park Road, Jawahar Nagar, Hyderabad, Telangana 500020
9821711605 | https://academy.forumias.com | admissions@forumias.academy | helpdesk@forumias.academy
Page 26 of 29

SFG 2022 | LEVEL 1 | Test #37 – Solutions |


Big data is a term applied to data sets whose size or type is beyond the ability of traditional relational
databases to capture, manage and process the data with low latency. Big data has one or more of the following
characteristics: high volume, high velocity or high variety. Artificial intelligence (AI), mobile, social and the
Internet of Things (IoT) are driving data complexity through new forms and sources of data.
Statement 1 is correct. Big Data simply refers to a large amount of data which is of structured, semi-
structured or unstructured nature. The data pool is so voluminous that it becomes difficult for an
organization to manage and process it using traditional databases and software techniques. Therefore, big
data not only implies the enormous amount of available data but it also refers to the entire process of
gathering, storing, and analyzing that data.
Statement 2 is correct. The Internet of Things, the Internet, and the rapid development of mobile
communication networks have spawned big data problems and have created problems of speed, structure,
volume, cost, value, security privacy, and interoperability. Traditional IT processing methods are impotent
when faced with big data problems, because of their lack of scalability and efficiency. Big Data problems need
to be solved by Cloud computing technology, while big data can also promote the practical use and
implementation of Cloud computing technology. There is a complementary relationship between them.
Statement 3 is correct. With a population of 1.2 billion, the relevance of Big Data becomes all the more
pronounced for India. Recently, NITI Aayog also echoed the idea of evidence-based policymaking guided by
Big Data. NITI Aayog is currently working on a plan in collaboration with the private players to develop the
‘National Data & Analytics Platform’, which will act as a single source of sectorial data for citizens,
policymakers, and researchers.
Knowledge Base:

Source: https://pib.gov.in/PressReleaseIframePage.aspx?PRID=1600370
https://www.sciencedirect.com/topics/computer-science/big-data-problem

Q.47) Consider the following statements with reference to the mitochondria:


1. Mitochondria are the power-generating units of the cell.
2. Mitochondria are found in nearly all eukaryotic cells.
3. Each mitochondrion is a single membrane bound structure.
Which of the statements given above is/are correct?
a) 1 and 2 only
b) 2 and 3 only
c) 1 and 3 only
d) 1, 2 and 3

Ans) a
Exp) Option a is correct.
Mitochondria are membrane-bound cell organelles (mitochondrion, singular) that generate most of the
chemical energy needed to power the cell's biochemical reactions. Chemical energy produced by the
mitochondria is stored in a small molecule called adenosine triphosphate (ATP). Mitochondria contain their
own small chromosomes.
Statement 1 is correct. Mitochondria are the power-generating units of the cell and are abundant where
energy-requiring processes take place, such as skeletal and cardiac muscle. The primary role of mitochondria
is to convert the products of carbohydrate, protein, and fat metabolism to CO2 and water, using key enzymes
of the electron transport chain.
Statement 2 is correct. Mitochondria are found in nearly all eukaryotic cells. Some cells may have single
large mitochondrion, but more often a cell has hundreds or even thousands of mitochondria at variable
location in cells depending upon the cell function. Mitochondria possess recognizable morphological
characteristics despite considerable variability in appearance. Typical mitochondria are sausage shaped. In
electron micrographs, they appear mostly as rod-shaped or cylindrical.

Forum Learning Centre: Delhi - 2nd Floor, IAPL House, 19 Pusa Road, Karol Bagh, New Delhi - 110005 | Patna - 2nd floor, AG Palace, E Boring Canal Road,
Patna, Bihar 800001 | Hyderabad - 1st & 2nd Floor, SM Plaza, RTC X Rd, Indira Park Road, Jawahar Nagar, Hyderabad, Telangana 500020
9821711605 | https://academy.forumias.com | admissions@forumias.academy | helpdesk@forumias.academy
Page 27 of 29

SFG 2022 | LEVEL 1 | Test #37 – Solutions |


Statement 3 is incorrect. Each mitochondrion is a double membrane bound structure i.e., outer and the
inner membrane, each consisting of phospholipid's bilayer with proteins. The outer membrane is smooth,
but the inner membrane has infolding’s called cristae (singular: crista) that provides it comparatively larger
surface area. The inner membrane divides the mitochondrion into two internal compartments. The narrow
region between the inner and outer membrane is the peri-mitochondrial space, and the innermost
compartment lined by inner membrane is
called mitochondrial matrix.
Source: https://ncert.nic.in/textbook/pdf/kebt102.pdf
https://www.ncbi.nlm.nih.gov/pmc/articles/PMC3951182/

Q.48) Which of the following is incorrect with reference to the QR (quick response) code and Bar code
functions:
a) QR code contains more information than a barcode.
b) QR code only contains information in a horizontal direction.
c) Unlike a normal barcode, QR code is two-dimensional.
d) QR code works even if the printed code is damaged or soiled.

Ans) b
Exp) Option b is correct.
A QR code (Quick Response code) is a type of matrix barcode (or two-dimensional barcode invented in 1994
by the Japanese automotive company Denso Wave. A barcode is a machine-readable optical label that can
contain information about the item to which it is attached.
Statement a is correct. QR code contains information in both a horizontal and a vertical direction, hence
the name “2-dimensional code.” Due to this structural difference, a QR code contains a hundred times more
information than a barcode and has a greater potential to store more information in a smaller space than a
barcode.
Statement b is incorrect. One of the biggest differences comes in terms of the differences between their
designs or simply how they look. This also applies to the design of the two codes. A barcode (and not QR
code) only contains information in a horizontal direction.
Statement c is correct. QR code, is similar to a barcode. Like barcode, it also contains machine-readable
information about the object it is attached to. Unlike a normal barcode, a QR code is two-dimensional, i.e.,
it contains information in both the vertical and horizontal directions.
Statement d is correct. Adaptation and error corrections are less in the QR codes. This is by far the most
desirable advantage of QR codes over conventional barcodes. QR codes have an error rate of 7-30%. Simply
put, even if the packaging of the product in question or the printed code is damaged or soiled, the QR code
works. This feature is smartly used by companies and businesses. Because of the error-correction feature,
they can (and do) put a small logo or a picture within the code to make it more associable to the business in
question.

QR code and Bar code


Source: https://www.scienceabc.com/innovation/whats-qr-code-how-its-different-from-
barcode.html#:~:text=Whereas%20a%20barcode%20only%20contains,has%20a%20greater%20potential%
20to

Q.49) Consider the following statements with reference to the Machine learning:
1. Machine learning is a subset of deep learning that relates the recurrent neural networks and artificial neural
networks together.
2. The primary aim of machine learning is to develop computer programs that access the required data and
utilize it for learning by themselves.
3. Machine learning plays a significant role in self-driving cars.
Which of the statements given above is/are correct?
a) 1 and 2 only
b) 2 and 3 only
c) 1 and 3 only
d) 1, 2 and 3

Forum Learning Centre: Delhi - 2nd Floor, IAPL House, 19 Pusa Road, Karol Bagh, New Delhi - 110005 | Patna - 2nd floor, AG Palace, E Boring Canal Road,
Patna, Bihar 800001 | Hyderabad - 1st & 2nd Floor, SM Plaza, RTC X Rd, Indira Park Road, Jawahar Nagar, Hyderabad, Telangana 500020
9821711605 | https://academy.forumias.com | admissions@forumias.academy | helpdesk@forumias.academy
Page 28 of 29

SFG 2022 | LEVEL 1 | Test #37 – Solutions |


Ans) b
Exp) Option b is correct.
Machine learning (ML) is a type of artificial intelligence (AI) that allows software applications to become more
accurate at predicting outcomes without being explicitly programmed to do so. Machine learning algorithms
use historical data as input to predict new output values.
Statement 1 is incorrect. Machine learning is an application and subset of AI (Artificial Intelligence) that
provides a system with the ability to learn from its experiences and improve accordingly without someone
physically programming those changes into it. Machine learning lets systems and devices improve without
being programmed to that particular level. It makes use of data for training so that it can find some accurate
results. Deep learning is basically a subset of machine learning that relates the recurrent neural networks
and artificial neural networks together. Its algorithms are exactly like machine learning.
Statement 2 is correct. The primary aim of machine learning is to develop computer programs that access
the required data and utilize it for learning by themselves. The process of learning begins with observations
or data, such as examples, direct experience, or instruction, in order to look for patterns in data and make
better decisions in the future based on the examples that we provide.
Statement 3 is correct. There are number of applications of the machine learning. One of the most exciting
applications of machine learning is self-driving cars. Machine learning plays a significant role in self-
driving cars. Tesla, the most popular car manufacturing company is working on self-driving car. It is using
unsupervised learning method to train the car models to detect people and objects while driving.

Knowledge Base: Other applications of machine learning:


 Image Recognition: Image recognition is one of the most common applications of machine learning. It
is used to identify objects, persons, places, digital images, etc. The popular use case of image recognition
and face detection is, Automatic friend tagging suggestion:
o Facebook provides us a feature of auto friend tagging suggestion. Whenever we upload a photo
with our Facebook friends, then we automatically get a tagging suggestion with name, and the
technology behind this is machine learning's face detection and recognition algorithm.
o It is based on the Facebook project named "Deep Face," which is responsible for face recognition
and person identification in the picture.
 Speech Recognition: While using Google, we get an option of "Search by voice," it comes under speech
recognition, and it's a popular application of machine learning.
o Speech recognition is a process of converting voice instructions into text, and it is also known
as "Speech to text", or "Computer speech recognition." At present, machine learning algorithms
are widely used by various applications of speech recognition. Google assistant, Siri, Cortana,
and Alexa are using speech recognition technology to follow the voice instructions.
 Traffic prediction: If we want to visit a new place, we take help of Google Maps, which shows us the
correct path with the shortest route and predicts the traffic conditions.
o It predicts the traffic conditions such as whether traffic is cleared, slow-moving, or heavily
congested with the help of two ways:
 Real Time location of the vehicle form Google Map app and sensors.
Source: https://www.expert.ai/blog/machine-learning-
definition/#:~:text=Machine%20learning%20is%20an%20application,it%20to%20learn%20for%20themsel
ves.
https://www.javatpoint.com/applications-of-machine-learning

Q.50) Which of the following is correct with reference to Hyper spectral imaging technology:
a) Hyper spectral imaging is a technique that analyzes a wide spectrum of light instead of assigning primary
colors to each pixel.

Forum Learning Centre: Delhi - 2nd Floor, IAPL House, 19 Pusa Road, Karol Bagh, New Delhi - 110005 | Patna - 2nd floor, AG Palace, E Boring Canal Road,
Patna, Bihar 800001 | Hyderabad - 1st & 2nd Floor, SM Plaza, RTC X Rd, Indira Park Road, Jawahar Nagar, Hyderabad, Telangana 500020
9821711605 | https://academy.forumias.com | admissions@forumias.academy | helpdesk@forumias.academy
Page 29 of 29

SFG 2022 | LEVEL 1 | Test #37 – Solutions |


b) Hyper spectral imaging has been particularly useful in satellite technology only.
c) Unlike optical technologies which can scan full color spectrum, Hyper spectral imaging can only scan for
a single color.
d) The spectral range in hyper spectral data cannot extend beyond the visible range.

Ans) a
Exp) Option a is correct.
Hyperspectral imaging, like other spectral imaging, collects and processes information from across the
electromagnetic spectrum. The goal of hyperspectral imaging is to obtain the spectrum for each pixel in the
image of a scene, with the purpose of finding objects, identifying materials, or detecting processes.
Statement a is correct. Hyper spectral imaging (HSI) is a technique that analyses a wide spectrum of light
instead of just assigning primary colours (red, green, blue) to each pixel. The light striking each pixel is
broken down into many different spectral bands in order to provide more information on what is imaged.
Statement b is incorrect. The algorithms and the image processing methodologies associated with HSI are
a product of military research, and were primarily used to identify targets and other objects against
background clutter. In the past, HSI has seen civil applications, and has particularly been useful in satellite
technology. It might become an inexpensive, promising, and quick tool for the assessment of tissue
conditions at diagnosis and during surgery. The medical applications include forensics, detection of
colorectal and gastric cancer.
Statement c is incorrect. In HSI, the unique colour signature of an individual object can be detected. Unlike
other optical technologies that can only scan for a single colour, HSI is able to distinguish the full colour
spectrum in each pixel. Therefore, it provides spectral information in addition to 2D spatial images.
Statement d is incorrect. Hyper spectral data cubes can contain absorption, reflectance, or fluorescence
spectrum data for each image pixel. It is assumed that HSI data is spectrally sampled at more than 20 equally
distributed wavelengths. The spectral range in hyper spectral data can extend beyond the visible range
(ultraviolet, infrared).
Source: https://www.sciencedirect.com/topics/medicine-and-dentistry/hyperspectral-
imaging#:~:text=Hyperspectral%20imaging%20(HSI)%20is%20a,%2C%20blue)%20to%20each%20pixel.&te
xt=Unlike%20other%20optical%20technologies%20that,color%20spectrum%20in%20each%20pixel.

Forum Learning Centre: Delhi - 2nd Floor, IAPL House, 19 Pusa Road, Karol Bagh, New Delhi - 110005 | Patna - 2nd floor, AG Palace, E Boring Canal Road,
Patna, Bihar 800001 | Hyderabad - 1st & 2nd Floor, SM Plaza, RTC X Rd, Indira Park Road, Jawahar Nagar, Hyderabad, Telangana 500020
9821711605 | https://academy.forumias.com | admissions@forumias.academy | helpdesk@forumias.academy

You might also like